162
TỔNG HỢP CÁC ĐỊNH LÍ HÌNH HỌC TRONG CÁC KÌ THI HỌC SINH GIỎI (Nguồn: Mathscope.org) Những kiến thức sau đây gồm một số kiến thức cơ sở để khám phá hình học olympiad hoặc là những kết quả đẹp nổi tiếng :hornytoro:.Bài viết này được soạn ra nhằm đáp ứng nhu cầu tra cứu ,học hỏi của nhiều bạn đọc. Nó sẽ cần sự chung tay của nhiều thành viên !. Đầu tiên mình sẽ giới thiệu mục lục và nếu ai biết phần kiến thức ấy thì có thể post lên , nhưng để đảm bảo cho tính hệ thống , chặt chẽ và dễ theo dõi của bài viết ,mình xin nêu một số quy ước như sau: 1) Mỗi bài viết đều phải vẽ hình minh họa. 2)Mỗi bài viết chỉ đề cập đến 1 đề mục kiến thức. 3) Phải đảm bảo thứ tự nêu trong mục lục. 4)Chúng tôi chỉ giữ lại những trao đổi có ích kể từ sau khi hoàn thành mục lục, điều đó có nghĩa là những trao đổi chen giữa không bị xóa lúc này nhưng sẽ bị xóa khi mục lục được hoàn tất.. Bây giờ sẽ là nội dung chính A/ MỤC LỤC I/ Một số định nghĩa ,định lí , điểm và đường đặc biệt không duy nhất : I.1)Định lí Menelaus I.2)Mở rộng định lí Menelaus theo diện tích I.3)Định lí Menelaus cho tứ giác I.4)Định lí Ceva I.5)Định lí Ceva dạng sin I.6)Định lí Desargues I.7)Định lí Pappus I.8)Một trường hợp đặc biệt của định lí Pappus qua góc nhìn hình xạ ảnh. I.9)Đẳng thức Ptolemy I.10)Bất đẳng thức Ptolemy I.11)Định lí Pascal I.12)Định lí Brianchon 1

Tong Hop Cac Dinh Li Hinh Hoc Phang Trong Cac Ki ThiHSG

Embed Size (px)

DESCRIPTION

gfg

Citation preview

Page 1: Tong Hop Cac Dinh Li Hinh Hoc Phang Trong Cac Ki ThiHSG

TỔNG HỢP CÁC ĐỊNH LÍ HÌNH HỌC TRONG CÁC KÌ THI HỌC SINH GIỎI

(Nguồn: Mathscope.org)Những kiến thức sau đây gồm một số kiến thức cơ sở để khám phá hình học olympiad hoặc là những kết quả đẹp nổi tiếng :hornytoro:.Bài viết này được soạn ra nhằm đáp ứng nhu cầu tra cứu ,học hỏi của nhiều bạn đọc.Nó sẽ cần sự chung tay của nhiều thành viên !. Đầu tiên mình sẽ giới thiệu mục lục và nếu ai biết phần kiến thức ấy thì có thể post lên , nhưng để đảm bảo cho tính hệ thống , chặt chẽ và dễ theo dõi của bài viết ,mình xin nêu một số quy ước như sau:1) Mỗi bài viết đều phải vẽ hình minh họa.2)Mỗi bài viết chỉ đề cập đến 1 đề mục kiến thức.3) Phải đảm bảo thứ tự nêu trong mục lục.4)Chúng tôi chỉ giữ lại những trao đổi có ích kể từ sau khi hoàn thành mục lục, điều đó có nghĩa là những trao đổi chen giữa không bị xóa lúc này nhưng sẽ bị xóa khi mục lục được hoàn tất..Bây giờ sẽ là nội dung chínhA/ MỤC LỤC

I/ Một số định nghĩa ,định lí , điểm và đường đặc biệt không duy nhất :

I.1)Định lí MenelausI.2)Mở rộng định lí Menelaus theo diện tíchI.3)Định lí Menelaus cho tứ giácI.4)Định lí Ceva I.5)Định lí Ceva dạng sinI.6)Định lí DesarguesI.7)Định lí PappusI.8)Một trường hợp đặc biệt của định lí Pappus qua góc nhìn hình xạ ảnh.I.9)Đẳng thức PtolemyI.10)Bất đẳng thức PtolemyI.11)Định lí PascalI.12)Định lí BrianchonI.13)Định lí MiquelI.14)Công thức CarnotI.15)Định lí CarnotI.16)Định lí BrokardI.17)Định lí Euler về khoảng cách giữa tâm 2 đường tròn nội, ngoại tiếp của tam giácI.18)Định lí Euler về khoảng cách giữa tâm 2 đường tròn nội, ngoại tiếp của tứ giác (Định lí Fuss)I.19)Định lí CaseyI.20)Định lí StewartI.21)Định lí Lyness I.22)Định lí Lyness mở rộng (Bổ đề Sawayama)I.23)Định lí Thébault

1

Page 2: Tong Hop Cac Dinh Li Hinh Hoc Phang Trong Cac Ki ThiHSG

I.24)Công thức Jacobi liên quan đến tâm tỉ cự,định lí LebnitzI.25)Định lí Newton cho tứ giác ngoại tiếpI.26)Định lí BreichneiderI.27)Định lí con nhímI.28)Định lí Gergonne -EulerI.29)Định lí PeletierI.30)Định lí MiobiutI.31)Định lí VivianiI.32)Công thức Lagrange mở rộngI.33) Đường thẳng SimsonI.34)Đường thẳng Steiner I.35) Điểm Anti-Steiner (Định lí Collings)I.36)Định lí NapoleonI.37)Định lí MorleyI.38)Định lí con bướm với đường trònI.39)Định lí con bướm với cặp đường thẳngI.40)Điểm BlaikieI.41)Định lí chùm đường thẳng đồng quyI.42)Đường tròn ApolloniusI.43)Định lí BlanchetI.44)Định lí Blanchet mở rộng I.45) Định lí JacobiI.46) Định lí KiepertI.47)Định lí KariyaI.48)Cực trực giaoI.49)Khái niệm tam giác hình chiếu ,công thức Euler về diện tích tam giác hình chiếuI.50)Khái niệm hai điểm đẳng giácI.51)Khái niệm tứ giác toàn phần.I.52)Đường thẳng Droz-FarnyI.53) Đường tròn Droz-FarnyI.54)Định lí Van Aubel về tứ giác và các hình vuông dựng trên cạnhI.55)Hệ thức Van AubelI.56)Định lí PithotI.57)Định lí JohnsonI.58) Định lí EyeballI.59) Bổ đề HarukiI.60)Bài toán LangleyI.61)Định lí Paul Yiu về đường tròn bàng tiếp.I.62)Định lí MaxwellI.63)Định lí Brahmagupta về tứ giác nội tiếp có hai đường chéo vuông góc.I.64)Định lí SchootenI.65)Định lí BottemaI.66)Định lí PompeiuI.67)Định lí ZaslavskyI.68)Định lí ArchimedesI.69) Định lí Urquhart

2

Page 3: Tong Hop Cac Dinh Li Hinh Hoc Phang Trong Cac Ki ThiHSG

I.70)Định lí Mairon WaltersI.71)Định lí Poncelet về bán kính đường tròn nội tiếp,bàng tiếp trong tam giác vuông.I.72)Định lí HansenI.73)Định lí Steinbart suy rộng I.74)Định lí Monge & d'Alembert II.75)Định lí Monge & d'Alembert III.76)Định lí Steiner về bán kính các đường tròn.I.77)Định lí BellavitisI.78)Định lí Feuer bach-Luchterhand:

II/Một số điểm và đường đặc biệt được xác định duy nhất với tam giác và tứ giác,tứ điểm:

Ở đây nếu không giải thích gì thêm thì yếu tố được hiểu là trong tam giác.

II.1) Đường thẳng Euler của tam giácII.2)Đường tròn và tâm Euler II.3)Đường đối trung, điểm LemoineII.4)Điểm Gergone,điểm Nobb, đường thẳng GergoneII.5)Điểm NagelII.6)Điểm Brocard II.7)Điểm SchifflerII.8)Điểm FeuerbachII.9)Điểm KosnitaII.10)Điểm Musselman,định lí Paul Yiu về điểm MusselmanII.11)Khái niệm vòng cực của tam giác.II.12)Điểm GibertII.13)Trục Lemoine II.14)Tâm MorleyII.15) Tâm Spieker và đường thẳng NagelII.16)Hai điểm FermatII.17)Điểm Parry reflection.II.18)Đường tròn Taylor ,tâm TaylorII.19)Điểm Bevan II.20)Điểm VectenII.21)Điểm MittenpunktII.22)Điểm NapoleonII.23)Đường tròn Adam

3

Page 4: Tong Hop Cac Dinh Li Hinh Hoc Phang Trong Cac Ki ThiHSG

II.24)Tam giác Fuhrmann ,đường tròn FuhrmannII.25)Hình luc giác và đường tròn Lemoine thứ nhấtII.26)Hình lục giác và đường tròn Lemoine thứ haiII.27)Điểm Euler của Tứ giác nội tiếpII.28)Đường thẳng Steiner của tứ giác toàn phầnII.29)Đường thẳng Gauss của tứ giác toàn phần.II.30) Điểm Miquel của tứ giác toàn phầnII.31)Đường tròn Miquel của tứ giác toàn phầnII.32)Hình bình hành Varignon của tứ giác .II.33)Điểm Poncelet của tứ giác.

III/Một số mảng kiến thức quan trọng.

III.1)Tỉ số kép, phép chiếu xuyên tâm III.2)Hàng điểm điều hòa và một số hệ thức liên quan ,III.3)Chùm điều hòa, tứ giác điều hòaIII.4)Góc giữa đường thẳng và đường tròn, giữa hai đường tròn, đường tròn trực giaoIII.5) Cực và đối cực

IV/Một số định lí không chứng minh

Ở đây sẽ giới thiệu một số định lí rất hay và dễ hiểu ( nhưng cách chứng minh mà mình biết là phức tạp ) tuy nhiên rất vui nếu ai đó sẽ giới thiệu những chứng minh của nó:hornytoro:

IV.1) Định lí AiyerIV.2)Đường tròn LesterIV.3)Tâm EppsteinIV.4)Đường tròn Neuberg-Mineur của tứ giácIV.5)Paracevian perspector

B/MỘT SỐ KHÁI NIỆM,ĐỊNH LÍ.

I.1)Định lí Menelaus

4

Page 5: Tong Hop Cac Dinh Li Hinh Hoc Phang Trong Cac Ki ThiHSG

Định lí: Cho tam giác ABC và 3 điểm M,N,P lần lượt thuộc BC,CA,AB.Khi đó M,N,P thẳng hàng khi và chỉ khi:

(1)Chứng minh:

a)Khi M,N,P thẳng hàng.Trên MN lấy 1 điểm Q sao cho AQ//BC Theo Thales ;

Từ đó dễ có đẳng thức (1)trên.b)Ngược lại ,khi có (1):Giả sử PN cắt BC tại M'.Theo phần trước ta có:

Kết hợp với (1) suy ra

Do đó M trùng M' tức là M,N,P thẳng hàng.

5

Page 6: Tong Hop Cac Dinh Li Hinh Hoc Phang Trong Cac Ki ThiHSG

Vậy ta có điều phải chứng minh.

(Xem them : eeg-11.bdf; ge_G1.bdf; 6-concur-solns.bdf)

I.2)Mở rộng định lí Menelaus theo diện tích

Định lí:Cho tam giác ABC và 3 điểm M,N,P lần lượt nằm trên BC,CA,AB.Khi đó ta có:

Chứng minh :(thamtuhoctro post)

Gọi là vector chỉ phương của Ta có:

mặt khác :

tương tự:

Ta suy ra:

6

Page 7: Tong Hop Cac Dinh Li Hinh Hoc Phang Trong Cac Ki ThiHSG

I.3)Định lí Menelaus cho tứ giác:

Định lí:Cho tứ giác ABCD và một đường thẳng d cắt AB,BC,CD,DA lần lượt ở M,N,P,Q. Khi đó ta có:

Chứng minh:Ta sẽ làm giống cách chứng minh ở tam giácTrên d lấy hai điểm I,J sao cho AI//BJ//CDTheo Thales ta có:

Từ đó dễ có điều cần chứng minh.

7

Page 8: Tong Hop Cac Dinh Li Hinh Hoc Phang Trong Cac Ki ThiHSG

*Chú ý 1)Khi áp dụng cho tứ giác ,định lí Menelaus chỉ phát biểu dạng thuận bởi dạng đảo nói chung không đúng!2) Các bạn thử suy nghĩ xem với dạng thuận như thế này thì có thể mở rộng cho đa giác được không? -Một vấn đề khá thú vị

I.4) Định lý Ceva

Định lý:Cho tam giác ABC.Gọi E, F, G là ba điểm tương ứng nằm trên BC, CA, AB. Ba đường thẳng AE, BF, CG cắt nhau tại một điểm O khi và chỉ khi:

Chứng minh:Phần thuận:Giả sử ba đường thẳng AE, BF, CG cắt nhau tại một điểm O. TỪ A và C, kẻ các đường song song với BF, chúng lần lượt cắt CG và AE tại K, I tương ứng.

Ta có: và (Sử dụng định lý Thales)

. Các cặp tam giác đồng dạng IEC và OEB, AKG và BOG : và

Do đó:

Phần đảo:

8

Page 9: Tong Hop Cac Dinh Li Hinh Hoc Phang Trong Cac Ki ThiHSG

Giả sử ta có: Qua giao điểm của các đường thẳng AE và BF, kẻ đường thẳng với nằm trên cạnh AB. Khi

đó, theo chứng minh phần thuận:

Suy ra , hay , ta có điều phải chứng minh

I.5) Định lý Ceva sin

Định lý: Gọi E, F, G là ba điểm tương ứng nằm trên các đường thẳng BC, CA, AB của tam giác ABC. Ba

đường thẳng AE, BF, CG cắt nhau tại một điểm O khi và chỉ khi:

Chứng minh:Phần thuận: Giả sử AE, BF, CG đồng quy tại O. Khi đó hai tam giác ABE và ACE có cùng chiều cao hạ từ đỉnh A.

Tương tự

Nhân từng vế ba đẳng thức trên được: (Theo định lý Ceva)Từ đó suy ra đpcm.

Phần đảo: CM tương tự phần đảo ở mục 4.

9

Page 10: Tong Hop Cac Dinh Li Hinh Hoc Phang Trong Cac Ki ThiHSG

I.6) Định lý Desargues

Định lý: Cho tam giác ABC và tam giác A'B'C'. Khi đó AA', BB', CC' đồng quy khi và chỉ khi các giao điểm của BC và B'C', CA và C'A', AB và A'B' thẳng hàng.

Chứng minh:

Gọi X, Y, Z là lần lượt là các giao điểm của các cặp cạnh BC và B’C’, CA và C’A’, AB và A’B’ .

Phần thuận:Giả sử các đường thẳng AA’, BB’, CC’ đồng quy tại S. Ta chứng minh X, Y, Z thẳng hàng.Áp dụng định lí Menelaus cho tam giác SBC với cát tuyến XB'C' ta có:

hay

Tương tự, ta có:

Nhân từng vế các đẳng thức trên lại với nhau, và theo định lí Menelaus suy ra X, Y, Z thẳng hàng.

10

Page 11: Tong Hop Cac Dinh Li Hinh Hoc Phang Trong Cac Ki ThiHSG

Phần đảo:Giả sử các điểm X, Y, Z thẳng hàng. Ta chứng minh các đường thẳng AA’, BB’, CC’ đồng quy.Gọi S là giao điểm của AA’ và BB’. SC cắt đường thẳng AC’ tại C”.Xét 2 tam giác ABC và A’B’C” có các đường nối các đỉnh tương ứng đồng quy, do đó theo phần thuận giao điểm của các cạnh tương ứng cũng đồng quy.Ta thấy AB cắt A’B’ tại Z, AC cắt A’C” tại Y (do A’, C’, C” thẳng hàng), suy ra giao điểm X’ của BC và B’C” phải thuộc YZ. Tức là X’ là giao của YZ và BC nên X’ trùng với X.Suy ra C” trùng với C’, hay AA’, BB’, CC’ đồng quy.

I.7)Định lí Pappus

Định lí: Cho ba điểm A,B,C nằm trên đường thẳng a, X,Y,Z nằm trên đường thẳng b.Gọi M,N,P lần lượt là giao điểm của các cặp đường thẳng (AY,BX) ,(AZ,CX),CY,BZ).Khi đó M,N,P thẳng hàng.

Chứng minh:

Định lí này có một cách chứng minh dùng Menelaus ,nếu có điều kiện mình sẽ post lên,còn sau đây là một cách dựa trên kiến thức cơ sở về tỉ số kép và phép chiếu xuyên tâm.Ta có bổ đề sau được chứng minh dễ dàng nhờ những hiểu biết ban đầu về tỉ số kép và phép chiếu xuyên tâm:

Bổ đề: Cho góc xOy và các điểm A,B,C thuộc Ox; D,E,F thuộc Oy.Khi đó AD,BE,CF đồng quy khi và chỉ khi: (OABC) =(ODEF) .

Bổ đề trên bạn đọc tự chứng minh, bây giờ ta sẽ trở lại bài toán.Kí hiệu là phép chiếu xuyên tâm E.

11

Page 12: Tong Hop Cac Dinh Li Hinh Hoc Phang Trong Cac Ki ThiHSG

Gọi T,Q lần lượt là giao điểm của BX và AZ; CX và BZ.Sử dụng bổ đề trên thì ta sẽ cần chứng minh: (BTMX) =(BZPQ)

+)Trường hợp a//b bạn đọc hãy chứng minh nhờ Thales

+)Khi a không song song với b.Gọi S là giao của a và b.Ta thấy:Với :Với Từ đó suy ra điều cần chứng minh.

I.8)Một trường hợp đặc biệt của định lí Pappus qua góc nhìn hình xạ ảnh.

Ở phần này chúng tôi chỉ dùng hình xạ ảnh để dẫn dắt đến kết quả còn nội dung định lí và cách chứng minh thì hoàn toàn phù hợp với kiến thức hình THCS!Ta có kết quả sau liên quan đến hình xạ ảnh: Các đường thẳng song song với nhau thì gặp nhau tại một điểm ở vô cực và ngược lại .Vận dụng vào định lí Pappus ở trên , cho các điểm A,B,C ra vô cực thì theo kết quả về hình xạ ảnh ta cóYM//ZN ( Vì YM,ZN cùng đi qua một điểm (A) ở vô cực )Tương tự thì :XN//YP,XM//ZP.Và khi ấy M,N,P vẫn thẳng hàng. Ta phát biểu lại được một định lí đơn giản và hữu dụng sau đây:

Định lí:Trên mặt phẳng cho ba điểm X,Y,Z thẳng hàng và ba điểm M,N,P thỏa mãn XN//YP,YM//ZN,XM//ZP.Khi đó ta cũng có M,N,P thẳng hàng.

Chứng minh:

12

Page 13: Tong Hop Cac Dinh Li Hinh Hoc Phang Trong Cac Ki ThiHSG

Trường hợp MP//XYZ thì đơn giản,bạn đọc tự chứng minh.Ta sẽ xét khi MP không song song với XYZ.Gọi S là giao điểm của MP với XYZ.Đường thẳng qua X song song với YP cắt MP ở N'. Bài toán sẽ được gải quyết nếu ta chứng minh được rằng ZN' // YM (Vì khi ấy N' trùng N).

Thật vậy,chú ý YP//XN', ZP//XM nên theo Thales ta có:

Đến đây theo Thales đảo ta suy ra ZN' //YM. Chứng minh được hoàn tất.!

I.9)Đẳng thức Ptolemy

Định lí Với tứ giác nội tiếp ABCD thì:AB.CD+AD.BC=AC.BD

Chứng minh:

13

Page 14: Tong Hop Cac Dinh Li Hinh Hoc Phang Trong Cac Ki ThiHSG

Lấy điểm E thuộc AC sao cho

đồng dạng

Tương tự đồng dạng

I.10) Bất đẳng thức Ptolemy

Định lý: Cho tứ giác ABCD. Khi đó có

Chứng minh: Lấy E nằm trong tứ giác ABCD sao cho

14

Page 15: Tong Hop Cac Dinh Li Hinh Hoc Phang Trong Cac Ki ThiHSG

Khi đó ~ hay .

Hơn nữa ~ hay Vậy ta có (đpcm).

KHÁM PHÁ ĐỊNH LÍ PTÔ-LÊ-MÊ

tác giả:Zai zaiI. Mở đầu:

Hình học là một trong những lĩnh vực toán học mang lại cho người yêu toán nhiều điều thú vị nhất và khó khăn nhất. Nó đòi hỏi ta phải có những suy nghĩ sáng tạo và tinh tế. Trong lĩnh vực này cũng xuất hiện ko ít những định lí, phương pháp nhằm nâng cao tính hiệu quả trong quá trình giải quyết các bài toán, giúp ta chinh phục những đỉnh núi ngồ ghề và hiểm trở . Trong bài viết này zaizai xin giới thiệu đến các bạn một vài điều cơ bản nhất về định lí Ptô-lê-mê trong việc chứng minh các đặc tính của hình học phẳng. Dù đã rất cố gắng nhưng bài viết sẽ không thể tránh khỏi những thiếu xót mong rằng các bạn sẽ cùng zaizai bổ sung và phát triển nó.

II, Nội dung - Lí thuyết:1. Đẳng thức Ptô-lê-mê:Cho tứ giác nội tiếp đường tròn . Khi đó:

Hình minh họa (hình 1)

Chứng minh:

Lấy thuộc đường chéo sao cho

15

Page 16: Tong Hop Cac Dinh Li Hinh Hoc Phang Trong Cac Ki ThiHSG

Khi đó xét và có: Nên đồng dạng với Do đó ta có:

. Lại có: và nên

Suy ra hay Từ và suy ra:

Vậy đẳng thức Ptô-lê-mê được chứng minh.

2, Bất đẳng thức Ptô-lê-mê:Đây có thể coi là định lí Ptô-mê-lê mở rộng bởi vì nó không giới hạn trong lớp tứ giác nội tiếp .

Định lí: Cho tứ giác . Khi đó:

Hình minh họa (hình 2)

Chứng minh:

Trong lấy điểm M sao cho:

Dễ dàng chứng minh: Cũng từ kết luận trên suy ra:

Áp dụng bất đẳng thức trong tam giác và các điều trên ta có:

Vậy định lí Ptô-lê-mê mở rộng đã được chứng minh.

3, Định lí Ptô-lê-mê tổng quát: Trong mặt phẳng định hướng cho đa giác nội tiếp đường tròn . M là một điểm thuộc cung (Không chứa )Khi đó:.Trong đó:

16

Page 17: Tong Hop Cac Dinh Li Hinh Hoc Phang Trong Cac Ki ThiHSG

Đây là một định lí ko dễ dàng chứng minh được bằng kiến thức hình học THCS. Các bạn có thể tham khảo phép chứng minh trong bài viết Định lí Ptô-lê-mê tổng quát của Tiến sĩ Nguyễn Minh Hà, ĐHSP , Hà Nội thuộc Tuyển tập 5 năm Tạp chí toán học và tuổi trẻ. III, Ứng dụng của định lí Ptô-lê-mê trong việc chứng minh các đặc tính hình học:

1, Chứng minh quan hệ giữa các đại lượng hình học:Mở đầu cho phần này chúng ta sẽ đến với 1 ví dụ điển hình và cơ bản về việc ứng dụng định lí Ptô-lê-mê.Bài toán 1: Cho tam giác đều có các cạnh bằng Trên lấy điểm di động, trên tia đối của tia lấy điểm di động sao cho . Gọi là giao điểm của và . Chứng minh rằng: ( Đề thi vào trường THPT chuyên Lê Quí Đôn, thị xã Đông Hà, tỉnh Quảng Trị, năm học 2005-2006)

Hình minh họa (hình 3)

Chứng minh:

Từ giả thiết suy ra Xét và có:

Lại có Từ:

Suy ra tứ giác nội tiếp được đường tròn.Áp dụng định lí Ptô-lê-mê cho tứ giác nội tiếp và giả thiết ta có:

(đpcm)

Đây là 1 bài toán khá dễ và tất nhiên cách giải này ko được đơn giản lắm.Vì nếu muốn sử dụng đẳng thức Ptô-lê-mê trong 1 kì thi thì có lẽ phải chứng minh nó dưới dạng bổ đề. Nhưng điều chú ý ở đây là ta chẳng cần phải suy nghĩ nhiều khi dùng cách trên trong khi đó nếu dùng cách khác thì lời giải có khi lại ko mang vẻ tường minh.

Bài toán 2: Tam giác vuông có . Gọi là một điểm trên cạnh là một điểm trên cạnh kéo dài về phía điểm sao cho . Gọi là một điểm trên cạnh sao cho nằm trên một đường tròn. là giao điểm thứ hai của với đường tròn ngoại tiếp . Chứng minh rằng:

17

Page 18: Tong Hop Cac Dinh Li Hinh Hoc Phang Trong Cac Ki ThiHSG

(Đề thi chọn đội tuyển Hồng Kông tham dự IMO 2000, HongKong TST 2000)

Hình minh họa: (hinh 4)

Chứng minh:Xét các tứ giác nội tiếp và ta có:

(cùng chắn các cung tròn)

Mặt khác Xét và có:

(do )

(do )Áp dụng định lí Ptô-lê-mê cho tứ giác nội tiếp ta có:

Từ suy ra:(đpcm)

Có thể thấy rằng bài 1 là tư tưởng đơn giản để ta xây dựng cách giải của bài 2. Tức là dựa vào các đại lượng trong tam giác bằng nhau theo giả thiết ta sử dụng tam giác đồng dạng để suy ra các tỉ số liên quan và sử dụng phép thế để suy ra điều phải chứng minh. Cách làm này tỏ ra khá là hiệu quả và minh họa rõ ràng qua 2 ví dụ mà zaizai đã nêu ở trên. Để làm rõ hơn phương pháp chúng ta sẽ cùng nhau đến với việc chứng minh 1 định lí bằng chính Ptô-lê-mê.

Bài toán 3: ( Định lí Carnot)Cho tam giác nhọn nội tiếp trong đường tròn và ngoại tiếp đường tròn Gọi

lần lượt là khoảng cách từ tới các cạnh tam giác. Chứng minh rằng:

18

Page 19: Tong Hop Cac Dinh Li Hinh Hoc Phang Trong Cac Ki ThiHSG

Hình minh họa (hinh 5)

Chứng minh:Gọi lần lượt là trung điểm của . Giả sử

Tứ giác nội tiếp, theo đẳng thức Ptô-lê-mê ta có:

Do đó: Tương tự ta cũng có :

Mặt khác:

Từ ta có:

Đây là 1 định lí khá là quen thuộc và cách chứng minh khá đơn giản. Ứng dụng của định lí này như đã nói là dùng nhiều trong tính toán các đại lượng trong tam giác. Đối với trường hợp tam giác đó không nhọn thì cách phát biểu của định lí cũng có sư thay đổi.

2, Chứng minh các đặc tính hình học:

Bài toán 1: Cho tam giác nội tiếp trong đường tròn và . Các đường thẳng tiếp xúc với đường tròn tại cắt nhau ở . Chứng minh rằng đi qua điểm chính giữa của cung

Hình minh họa(hinh 6)

19

Page 20: Tong Hop Cac Dinh Li Hinh Hoc Phang Trong Cac Ki ThiHSG

Chứng minh:Gọi giao điểm của với đường tròn là . Nối .

Xét và có: chung

Tương tự ta cũng có Mặt khác ( do là 2 tiếp tuyến của đường tròn cắt nhau)

Nên từ Áp dụng định lí Ptô-lê-mê cho tứ giác nội tiếp ta có:

Từ .Vậy ta có điều phải chứng minh.

Đây có lẽ là một trong những lời giải khá là ngắn và ấn tượng của bài này.Chỉ cần qua vài quá trình tìm kiếm các cặp tam giác đồng dạng ta đã dễ dàng đi đến kết luận của bài toán. Tư tưởng ban đầu khi làm bài toán này chính là dựa vào lí thuyết trong cùng một đường tròn hai dây bằng nhau căng hai cung bằng nhau. Do có liên quan đến các đại lượng trong tứ giác nội tiếp nên việc chứng minh rất dễ dàng.

Bài toán 2: Cho tam giác ABC có I là tâm đường tròn nội tiếp, O là tâm đường tròn ngoại tiếp

và trọng tâm G. Giả sử rằng . Chứng minh rằng song song với .

Hình minh họa (hinh 7)

20

Page 21: Tong Hop Cac Dinh Li Hinh Hoc Phang Trong Cac Ki ThiHSG

Kéo dài cắt tại . Khi đó là điểm chính giữa cung (không chứa ).Ta có: . Lại có :

Do suy ra sđ cung Từ Áp dụng định lí Ptô-lê-mê cho tứ giác nội tiếp ta có:

Từ Áp dụng tính chất đường phân giác trong tam giác và (5) ta có:

Vậy

Mặt khác G là trọng tâm của tam giác suy ra

Từ Suy ra IG là đường trung bình của tam giác hay song song với .

Đây là một bài toán khá là hay ít nhất là đối với THCS và với cách làm có vẻ "ngắn gọn" này ta đã phần nào hình dung được vẻ đẹp của các định lí.

Bài toán 3:Cho tam giác ABC nội tiếp đường tròn (O), CM là trung tuyến. Các tiếp tuyến tại A và B của (O) cắt nhau ở D. Chứng minh rằng:

Hình minh họa hinh 8)

21

Page 22: Tong Hop Cac Dinh Li Hinh Hoc Phang Trong Cac Ki ThiHSG

Chứng minh:Gọi N là giao điểm của CD với (O). Xét tam giác DNB và DBC có:

chung.

Tương tự ta cũng có :

Mà nên từ Áp dụng định lí Ptô-lê-mê cho tứ giác nội tiếp ta có:

Từ (3) và giả thiết

Xét và có:

Vậy bài toán được chứng minh.

Cơ sở để ta giải quyết các bài toán dạng này là tạo ra các tứ giác nội tiếp để áp dụng định lí sau đó sử dụng lí thuyết đồng dạng để tìm ra mối quan hệ giữa các đại lượng. Đây là một lối suy biến ngược trong hình học.

3, Chứng minh các đẳng thức hình học:

Bài toán 1: Giả sử là các điểm nằm trong sao cho

. Chứng minh rằng:

Hình minh họa: (hinh 9)

Chứng minh:

22

Page 23: Tong Hop Cac Dinh Li Hinh Hoc Phang Trong Cac Ki ThiHSG

Lấy điểm K trên đường thẳng BN sao cho , lúc đó suy ra:

Mặt khác dễ thấy rằng , từ đó dẫn đến

.Cũng từ ta có:

.suy ra tứ giác nội tiếp đường tròn. Áp dụng định lí Ptô-lê-mê cho tứ giác ta có:

Nhưng từ và thì :

Nên ta có đẳng thức (3)

Đây là 1 trong những bài toán khá là cổ điển của IMO Shortlist. Ta vẫn có thể giải quyết bài toán theo một hướng khác nhưng dài và phức tạp hơn đó là sử dụng bổ đề: Nếu M,N là các

điểm thuộc cạnh BC của sao cho thì . Đây là một bổ đề mà các bạn cũng nên ghi nhớ.

Bài toán 2: Cho tứ giác ABCD nội tiếp trong đường tròn (O). Chứng minh rằng:

Hình minh họa hinh 10)

Chứng minh:Lấy E và F thuộc đường tròn sao cho:

Khi đó: Áp dụng định lí Ptô-lê-mê cho hai tứ giác nội tiếp AECD và BCDF ta có:

Mặt khác:

Do đó:

Suy ra: Từ (1), (2), (3) ta có điều phải chứng minh.

Bài toán 3: Cho tam giác ABC với BE, CF là các đường phân giác trong. Các tia EF, FE cắt đường tròn ngoại tiếp tam giác theo thứ tự tại M và N. Chứng minh rằng:

Hình minh hoạ (hình 11)

23

Page 24: Tong Hop Cac Dinh Li Hinh Hoc Phang Trong Cac Ki ThiHSG

Chứng minh:Đặt

Áp dụng định lí Ptô-lê-mê cho hai tứ giác nội tiếp và ta có:

Từ (1) và (2) ta được:

Mặt khác ta lại có:

Tương tự :

Từ (4), (5) và tính chất đường phân giác ta có:

Chứng minh tương tự ta được:

Từ (3), (6), (7) ta có điều phải chứng minh.

Có thể dễ dàng nhận ra nét tương đồng giữa cách giải của 3 bài toán đó là vận dụng cách vẻ hình phụ tạo ra các cặp góc bằng các cặp góc cho sẵn từ đó tìm ra các biểu diễn liên quan. Một đường lối rất hay được sử dụng trong các bài toán dạng này.

4, Chứng minh bất đẳng thức và giải toán cực trị trong hình học:

Bài toán 1: (Thi HSG các vùng của Mĩ, năm 1987)

Cho một tứ giác nội tiếp có các cạnh liên tiếp bằng và các đường chéo bằng Chứng minh rằng:

Chứng minh:Áp dụng định lí Ptô-lê-mê cho tứ giác nội tiếp thì

Vậy ta cần chứng minh Bất đẳng thức này chính là một bất đẳng thức rất quen thuộc mà có lẽ ai cũng biết đó là bất

24

Page 25: Tong Hop Cac Dinh Li Hinh Hoc Phang Trong Cac Ki ThiHSG

đẳng thức Bunhiacopxki-BCS. Vậy bài toán được chứng minh.

Một lời giải đẹp và vô cùng gọn nhẹ cho 1 bài toán tưởng chừng như là khó. Ý tưởng ở đây là đưa bất đẳng thức cần chứng minh về 1 dạng đơn giản hơn và thuần đại số hơn. Thật thú vị là bất đẳng thức đó lại là BCS.

Bài toán 2:Cho lục giác lồi ABCDEF thỏa mãn điều kiện Chứng minh rằng:

HÌNH MINH HỌA (hinh 12)

Chứng minh:Đặt Áp dụng định lí Ptô-lê-mê mở rộng cho tứ giác ta có:

. Vì nên suy ra:

Tương tự ta cũng có:

Từ đó suy ra

Bất đẳng thức đã qui về dạng chính tắc SOS :

Dễ thấy:

Như vậy , đánh giá tương tự ta cũng dễ dàng thu được kết quả .Vậy bất đẳng thức được chứng minh. Đẳng thức xảy ra khi và chỉ khi .Tức là khi ABCDEF là một lục giác đều nội tiếp.

Đây là một bài toán do zaizai phát triển từ một bài toán quen thuộc. Nó cũng xuất phát từ bài Stronger than Nesbit inequality của mình. :lol:Cơ sở khi giải bài toán này là sử dụng phương pháp SOS để làm mạnh bài toán.Với bước chuyển từ việc chứng minh 1 bất đẳng thức hình học sang bất đẳng thức đại số ta dễ dàng tìm ra 1 lời giải đẹp. Nếu chuẩn hóa bất đẳng thứ này ta cũng có kết quả rất thú vị.

25

Page 26: Tong Hop Cac Dinh Li Hinh Hoc Phang Trong Cac Ki ThiHSG

Bài toán 3:Cho lục giác lồi ABCDEF thỏa mãn điều kiện và tổng độ dài ba cạnh bằng Chứng minh rằng:

Lời giải:Ta chuyển việc chứng minh bất đẳng thức trên về chứng minh bất đẳng thức sau:

Bằng cách sử dụng phương pháp hệ số bất định ta dễ dàng tìm được bất đẳng thức phụ đúng:

Tương tự với các phân thức còn lại ta có điều phải chứng minh.

Khi định hướng giải bài này chắc hẳn bạn sẽ liên tưởng ngay đến SOS nhưng thật sự thì nó ko cần thiết trong bài toán này bởi chỉ làm phức hóa bài toán. Dùng phương pháp hệ số bất định giúp ta tìm ra 1 lời giải ngắn và rất đẹp. Tuy nhiên lời giải này ko dễ hiểu lắm đối với THCS.

Thực ra cách làm mới bài toán này cũng cực kì đơn giản vì xuất phát điểm của dạng chuẩn là bất đẳng thức Nesbit quen thuộc vì vậy dễ dàng thay đổi giả thiết để biến đổi bài toán. Mà cách thay đổi điều kiện ở đây chính là bước chuẩn hóa trong chứng minh bất đẳng thức đại số. Nói chung là dùng để đồng bậc bất đẳng thức thuần nhất. Với tư tưởng như vậy ta hoàn toàn có thể xây dựng các kết quả mạnh hơn và thú vị hơn qua một vài phương pháp như SOS, hệ số bất định, dồn biến và chuẩn hóa. Đặc biệt sau khi chuẩn hóa ta có thể dùng 3 phương pháp còn lại để chứng minh.

Bài toán 4::Cho đường tròn và là một dây cung khác đường kính của đường tròn. Tìm điểm thuộc cung lớn sao cho lớn nhất.

Lời giải:Gọi D là điểm chính giữa cung nhỏ BC.Đặt không đổi. Theo định lí Ptô-lê-mê ta có:

Do và ko đổi nên lớn nhất khi và chỉ khi lớn nhất khi và chỉ khi là điểm đối xứng của qua tâm của đường tròn.

IV, Bài tập:Bài 1 CMO 1988, Trung Quốc)

là một tứ giác nội tiếp với đường tròn ngoại tiếp có tâm ) và bán kính . Các tia

cắt lần lượt tại . Chứng minh rằng:

Bài 2:Cho đường tròn và dây cung khác đường kính. Tìm điểm A thuộc cung lớn của đường tròn để đạt giá trị lớn nhất.Bài 3:

Cho tam giác ABC nội tiếp đường tròn . Đường tròn nằm trong (O) tiếp xúc với (O) tại

26

Page 27: Tong Hop Cac Dinh Li Hinh Hoc Phang Trong Cac Ki ThiHSG

T thuộc cung AC (ko chứa B). Kẻ các tiếp tuyến tới . Chứng minh rằng:

Bài 4:Cho luc giác có các cạnh có độ dài nhỏ hơn 1. Chứng minh rằng trong ba đường chéo có ít nhất một đường chéo có độ dài nhỏ hơn .Bài 5:Cho hai đường tròn đồng tâm, bán kính của đường tròn này gấp đôi bán kính của đường tròn kia. là tứ giá nội tiếp đường tròn nhỏ. Các tia lần lượt cắt đường tròn

lớn tại .Chứng minh rằng: chu vi tứ giác lớn hơn 2 lần chu vi tứ giác .

I.11) Định lý Pascal

Định lý: Cho 6 điểm A,B,C,D,E,F cùng thuộc một đường tròn. Khi đó các giao điểm của các cặp cạnh AB và DE, BC và EF, CD và FA thẳng hàng.

Chứng minh: Gọi P,M,N lần lượt là giao điểm của AF và CD, AB và DE, BC và EF. Gọi P', M', N' lần lượt là giao điểm của BC và DE, BC và AF, DE và AF.Áp dụng định lí Menelaus cho P'M'N' với cát tuyến PCD:

Tương tự ta có:

và Nhân các biểu thức trên lại kết hợp với các biểu thức phương tích sau:

Ta có :

27

Page 28: Tong Hop Cac Dinh Li Hinh Hoc Phang Trong Cac Ki ThiHSG

.Áp dụng định lí Menelaus đảo ta có đpcm.

Các bạn có thể vào đây xem thêm:http://www.math.ust.hk/mathematical_...bur/v10_n3.pdf

I.12) Định lý Brianchon

Định lý: Cho lục giác ABCDEF ngoại tiếp (O). Chứng minh rằng ba đường chéo lớn AD, BE, CF đồng quy.

Chứng minh: Ta kí hiệu các tiếp điểm của (O) trên AB,BC,CD,DE,EF,FA lần lượt là M,N,P,Q,R,S. Xét cực và đối cực đối với (O). Gọi K,I,J lần lượt là giao điểm của các cặp đường thẳng (SM,PQ) ,(MN,QR),(NP,RS). Vì SM và PQ là đường đối cực của A và D nên AD là đường đối cực của K. Tương tự BE và FC lần lượt là đường đối cực của I và J. Dùng định lí Pascal cho lục giác nội tiếp MNPQRS ta có I,J,K thẳng hàng. Nên ta có các đường đối cực của I,J,K (lần lượt là BE,CF,AD) cùng đi qua cực của đường thẳng này (đường thẳng đi qua I,J,K) nên AD,BE,CF đồng quy (đpcm).Tương tự ngược lại có thế chứng minh định lí pascal thông qua Brianchon và cực đối cực(xem thêm cực đối cực ở mục III.5 hoặc xem http://mathscope.org/forum/showthread.php?t=5024).

I.13)Định lí Miquel

Định lí: Cho tam giác ABC và ba điểm M,N,P lần lượt nằm trên BC,CA,AB. Khi đó các đường tròn ngoại tiếp các tam giác APN,BPM và CMN đồng quy.

Chứng minh:

28

Page 29: Tong Hop Cac Dinh Li Hinh Hoc Phang Trong Cac Ki ThiHSG

Gọi S là giao điểm của (BPM) và (CMN).Ta sẽ chứng minh S nằm trên (APN).Thật vậy:

Suy ra điều cần chứng minh.!

I.14) Công thức Carnot

Định lý: Cho ABC nội tiếp (O,R). Gọi x,y,z lần lượt là khoảng cách từ O đến BC,AC,AB. Gọi r là bán kính đường tròn nội tiếp tam giác ABC. Ta có:a)Nếu nhọn thì công thức carno là .b)Nếu thì công thức carno là Chứng minh:

a)Nếu nhọn

29

Page 30: Tong Hop Cac Dinh Li Hinh Hoc Phang Trong Cac Ki ThiHSG

Gọi F, E, D lần lượt là trung điểm của BC,CA,AB. Như vậy ta có. Đặt .

Áp đụng bất đẳng thức Ptolemy cho tứ giác nội tiếp OFBD ta có:

hay

Tương tự ta có và

ta lại có Cộng bốn biểu thức trên lại ta có

b)Nếu chứng minh tương tự.

Viết dưới dạng lượng giác, công thức Carnot chính là hệ thức . Chú ý hệ thức này đúng với mọi tam giác.

I.15) Định lí Carnot

Định lý:

Cho . Gọi lần lượt là các điểm thuộc các cạnh . lần

lượt là các đường thẳng đi qua và vuông góc với . đồng quy khi và chỉ khi

30

Page 31: Tong Hop Cac Dinh Li Hinh Hoc Phang Trong Cac Ki ThiHSG

Chứng minh: a)Phần thuận:

Gọi đồng quy tại O.ĐPCM

Đẳng thức này đúng nên ta có điều phải chứng minh.b) Phần đảo

Gọi giao điểm của tại O. Qua O hạ đường vuông góc xuống AB tại P'. Áp dụng định lí

thuận ta có P trùng với P'đồng quy.

I.16/Định lý Brokard

Định lý:Cho tứ giác lồi ABCD nội tiếp đường tròn tâm O.AD giao BC tại M,AB giao CD tại N,AC giao BD tại I.Chứng minh rằng O là trực tâm của tam giác MIN.

31

Page 32: Tong Hop Cac Dinh Li Hinh Hoc Phang Trong Cac Ki ThiHSG

Chứng minh:

Gọi H là giao thứ 2 của hai đường tròn ngoại tiếp các tam giác AID,BIC.Xét tứ giác DOHC,ta có:

Từ đó suy ra tứ giác DOHC nội tiếp.Tương tự ta cũng suy ra tứ giác AOHB nội tiếp.Dễ thấy suy ra N nằm trên trục đẳng phương của hai đường tròn

--> thẳng hàng.Ta có:

Từ đó suy ra Tương tự ta có:Suy ra O là trực tâm tam giác MIN (đpcm)

I.17) Định lí Euler về khoảng cách giữa tâm 2 đường tròn nội, ngoại tiếp của tam giác

Định lý:Cho tam giác ABC nội tiếp (O;R) và ngoại tiếp (I;r). Chứng minh rằng .

32

Page 33: Tong Hop Cac Dinh Li Hinh Hoc Phang Trong Cac Ki ThiHSG

Chứng minh:Kéo dài AI cắt (O) tại M. Vẽ đường kính MN của đường tròn (O).Hạ . Kéo dài OI cắt (O) tại E và F. Ta có ~

. Mặt khác dễ dàng chứng minh Lại có nên ta có điều phải chứng minh.  

I.18)Định lí Euler về khoảng cách giữa tâm hai đường tròn nội ngoại tiếp tứ giác!(Định lí Fuss)

Định lí :Cho tứ giác ABCD vừa nội tiếp (O,R) vừa ngoại tiếp (I,r). Đặt d=OI. Khi đó ta có:

Chứng minh

33

Page 34: Tong Hop Cac Dinh Li Hinh Hoc Phang Trong Cac Ki ThiHSG

Gọi tiếp điểm của (I) trên AB,BC,CD,DA lần lượt là M,N,P,Q.BI,CI cắt (O) lần lượt ở E,F .

Ta thấy:

Do đó E,O,F thẳng hàng ,nên O là trung điểm của EF.Theo công thức đường trung tuyến trong tam giác IEF ta có:

Từ đó suy ra:

(vì  

I.19)Định lí Casey(Định lí Ptolemy mở rộng)

Định lí :Cho tứ giác ABCD nội tiếp (O,R). Đặt các đường tròn là các đường tròn tiếp xúc

với (O) tại các đỉnh A,B,C,D. Đăt là độ dài đoạn tiếp tuyến chung của hai đường tròn .

Trong đó là độ dài tiếp tuyến chung ngoài nếu hai đường tròn cùng tiếp xúc trong hoặc cùng tiếp xúc ngoài với (O), và là độ dài đoạn tiếp xúc trong nếu trong trường hợp còn lại. Các

đoạn , ... được xác định tương tự. Khi đó ta có:

Chứng minh

Ta chứng minh trường hợp cùng tiếp xúc ngoài với (O). Các trường hợp còn lại chứng minh tương tự. Lần lượt đặt tâm các đường tròn trên là A',B',C',D' và bán kính lần lượt là x,y,z,t.Đặt AB=a, BC=b, CD=c, DA=d, AC=m, BD=n. Áp dụng định lý Pythagore:

Mặt khác lại có: (theo định lí hàm số cos)

34

Page 35: Tong Hop Cac Dinh Li Hinh Hoc Phang Trong Cac Ki ThiHSG

Tương tự với , ... Ta có

(định lý Ptolemy)Ngược lại ta thấy định lý Ptolemy là một trường hợp đặc biệt của định lí Casey khi x=y=z=t=0.

I.20)Hệ thức Stewart

Định lí:Cho ba điểm A,B,C thẳng hàng. Và một điểm M bất kì. Ta luôn có hệ thức

Chứng minhQua M hạ .Ta có:

(Đưa về trường hợp hệ thức Stewart cho 4 điểm thẳng hàng (khi M nằm trên đường thẳng chứa A,B,C))

Ta có đpcm.

I.21)Định lí LynessĐịnh lí:Nếu đường tròn tâm O tiếp xúc trong với đường tròn ngoại tiếp tam giác ABC tại T và tiếp xúc với các cạnh AB,AC của tam giác lần lượt tại E và F thì tâm đường tròn nội tiếp của tam giác nằm trên EF.

35

Page 36: Tong Hop Cac Dinh Li Hinh Hoc Phang Trong Cac Ki ThiHSG

Chứng minh:Để chứng minh định lí này ta cần chứng minh 2 bổ đề sau:Bổ đề 1:AB là dây của một đường tròn tâm (O). Đường tròn (l) tiếp xúc với dây AB tại K và tiếp xúc trong với (O) tại T. Chứng minh L là trung điểm của cung AB ko chứa T và Bổ đề 2: Điểm M là trung điểm cung BC ko chứa A của đường tròn ngoại tiếp tam giác ABC. Điểm I thuộc đoạn MA sao cho MI=MB. Chứng minh rằng I là tâm đường tròn nội tiếp tam giác ABC.Việc chứng minh 2 bổ đề này là khá đơn giản.Ta tiếp tục quay trở lại với việc chứng minh định lí Lyness.kẻ TF giao (O) tại P; BP cắt EF tại H.Theo bổ đề 1 ta có BP là phân giác của góc B.

Ta có: nt

Theo bổ đề 1 ta lại có Theo bổ đề 2 ta được H là tâm đường tròn nội tiếp tam giác ABC (ĐPCM)

I.22)Định lý Lyness mở rộng(Bổ đề Sawayama)

Định lí:Cho tam giác ABC nội tiếp đường tròn (O).M thuộc BC (Có cách phát biểu khác là: cho tứ giác ABDC và M là giao của BC và AD; nhưng hai cách phát biểu này là tương đương). Một đường tròn (O') tiếp xúc với hai cạnh MA và MC tại E và F đồng thời tiếp xúc với cả đường tròn (O) tại K. Khi đó ta có tâm đường tròn nội tiếp của tam giác ABC nằm trên đường thẳng EF.

36

Page 37: Tong Hop Cac Dinh Li Hinh Hoc Phang Trong Cac Ki ThiHSG

Chứng minhKF cát đường tròn (O) tại G. Áp dụng bổ đề 1 tại bài viết của chu t tung về định lý Lyness ở trên, ta có G là điểm chính giữa cung BC. Gọi I là giao của AG với EF. Ta có

nội tiếp

~

~

Lại cũng theo bổ đề 1 ta có I là tâm nội tiếp của (theo bổ đề 2)Xem thêm các hệ quả của định lý Lyness tại báo toán tuổi thơ 2 số 42 và 43  

I.23) Định lí ThébaultĐịnh lí: Cho tam giác nội tiếp đường tròn . là một điểm nằm trên cạnh . Đường

tròn tâm tiếp xúc với 2 đoạn và tiếp xúc trong với . Đường tròn tâm tiếp xúc

với 2 đoạn và tiếp xúc trong với . Gọi là tâm nội tiếp tam giác . Ta có: thẳng hàng.

Chứng minh

37

Page 38: Tong Hop Cac Dinh Li Hinh Hoc Phang Trong Cac Ki ThiHSG

Gọi lần lượt là tiếp điểm của với . Gọi là giao điểm của và . Theo định lí lyness mở rộng(đã có trong bài của trung anh), là tâm nội tiếp tam giác . Vậy ta chỉ cần chứng minh thẳng hàng. Thật vậy, gọi lần lượt là giao điểm của

và ; và . Áp dụng định lí Thales ta có: . Vậy , thẳng hàng(dpcm)

I.24)Công thức Jacobi liên quan đến tâm tỉ cự,định lí Lebnitz

1)Công thức Jacobi:

Nếu I là tâm tỉ cự của hệ điểm ứng với các hệ số thì với mọi điểm M trên mặt phẳng ta đều có:

Chứng minh:

(do I là tâm tỉ cự của hệ điểm nên )->đpcm.

2)Định lí Lebnitz Đây là trường hợp đặc biệt của công thức trên khi n=3

3)Hệ quả khác:

Giá trị nhỏ nhất của biểu thức (với các kí hiệu như phần trên) đạt được khi .(phần này thuần về tính toán nên chắc không cần vẽ hình phải không anh ma29?)

I.25)Định lí Newton cho tứ giác ngoại tiếp

Định lýCho tứ giác ABCD ngoại tiếp đường tròn O.Khi đó trung điểm hai đường chéo AC,BD và tâm O thẳng hàng.

Hình: (vẽ bằng Cabri hơi xấu):rokeyrulez:

38

Page 39: Tong Hop Cac Dinh Li Hinh Hoc Phang Trong Cac Ki ThiHSG

Chứng minh

Gọi P,Q,R,S lần lượt là các tiếp điểm của các đoạn thẳng AB,BC,CD,DA đối với đường tròn .

Đặt .Áp dụng định lý con nhím cho tứ giác ABCD ta có:

<->

<->

<->

Từ đó suy ra hai vecto cùng phương->O,M,N thẳng hàng (đpcm)  

I.26)Định lí Breichneider (định lý hàm số cos cho tứ giác)

Định lýCho tứ giác ABCD có độ dài các cạnh AB, BC, CD, DA lần lượt là a, b, c, d và độ dài hai đường chéo

AC, BD là m, n. Khi đó ta có:

39

Page 40: Tong Hop Cac Dinh Li Hinh Hoc Phang Trong Cac Ki ThiHSG

Chứng minh

Trên cạnh AB ra phía ngoài dựng tam giác ABN đồng dạng với tam giác CAD, và dựng ra phía ngoài cạnh AD tam giác ADM đồng dạng với tam giác CAB. Khi đó dễ thấy:

và BDMN là hình bình hành.

Đồng thời có Áp dụng đính lí hàm số cos cho tam giác NAM, ta có

 

I.27)Định lí con nhím:

Định lí:Cho đa giác lồi và các vecto là các vecto có độ dài bằng các cạnh ,tương ứng vuông góc với các cạnh ấy và hướng ra phía ngoài đa giác.Thế thì :

40

Page 41: Tong Hop Cac Dinh Li Hinh Hoc Phang Trong Cac Ki ThiHSG

Chứng minh:Không giảm tính tổng quát ,giả sử đa giác có hướng dương.Xét phép quay vecto:

Từ đó suy ra điều cần chứng minh.

I.28)Định lí Gergone-Euler.

Định lí:Xét tam giác ABC và một điểm S trong mặt phẳng.AS,BS,CS lần lượt cắt BC,CA,AB ở D,E,F.Khi đó ta có:

Chứng minh:

41

Page 42: Tong Hop Cac Dinh Li Hinh Hoc Phang Trong Cac Ki ThiHSG

Kí hiệu chỉ diện tích đại số của đa giác.Ta thấy :

(1)Tương tự ta có:

(2)

(3)

Cộng theo vế (1),(2)và (3) ta có điều cần chứng minh.

I.29)Định lí Peletier

I.30)Định lí Miobiut

Định lí:Cho ngũ giác lồi

Chứng minh:

I.31)Định lí Viviani

Định lí: Trong tam giác đều ABC ta lấy 1 điểm S .Ta sẽ có tổng các khoảng cách từ điểm S tới ba cạnh sẽ có độ dài bằng 1 đường cao của tam giác.

Chứng minh:

42

Page 43: Tong Hop Cac Dinh Li Hinh Hoc Phang Trong Cac Ki ThiHSG

Kẻ SD,SE,SF lần lượt vuông góc với BC,CA,AB.Đặt độ dài cạnh tam giác ABC là a,độ dài đường cao AH là b.Ta có :

Giản ước hai vế cho a ta có điều cần chứng minh.  

I.32)Công thức Lagrange mở rộng.

Định lý:

Gọi I là tâm tỉ cự của hệ điểm ứng với các hệ số thì với mọi điểm M:

Chứng minh:

Từ hệ thức Jacobi (có thể xem ở mục I.24) thì ta chỉ cần chứng minh rằng:

Do I là tâm tỉ cự của hệ điểm nên:

<->

<->

43

Page 44: Tong Hop Cac Dinh Li Hinh Hoc Phang Trong Cac Ki ThiHSG

<->

<-> (đpcm)  

I.33) Đường thẳng SimsonĐịnh lí:Cho và điểm nằm trên đường tròn ngoại tiếp tâm của tam giác. Gọi

lần lượt là hình chiếu vuông góc của M trên các đường thẳng thì chúng cùng thuộc một đường thẳng (đây gọi là đường thẳng Simson).Chứng minh:

Dùng góc định hướngTa có:

Vậy thẳng hàng.

(Xem them riegelmj[1].bdf; 00045.bdf)

I.34) Đường thẳng Steiner

44

Page 45: Tong Hop Cac Dinh Li Hinh Hoc Phang Trong Cac Ki ThiHSG

Định lí:Cho và điểm nằm trên đường tròn ngoại tiếp tâm của tam giác. Gọi lần lượt là điểm đối xứng với của D qua các đường thẳng thì chúng cùng

thuộc một đường thẳng và đường thẳng này đi qua trực tâm H của tam giác ABC. Đường thẳng đó được gọi là đường thẳng steiner ứng với điểm D của tam giác ABC. Còn điểm D được gọi là điểm anti steiner.

Chứng minh:

Dễ thấy nếu gọi lần lượt là hình chiếu của D xuống ba cạnh của tam giác ABC thì là trung điểm của đoạn và tương tự ta có thẳng hàng.Ta có

(mod )Vậy đường thẳng steiner đi qua H.Từ đó ta có được tính chất rằng đường thẳng simson ứng với điểm D đi qua trung điểm của đoạn DH.

I.35) Điểm Anti Steiner(Định lí Collings)

Định lí:Cho và đường thẳng đi qua H trực tâm của tam giác ABC . Gọi lần lượt là đường thẳng đối xứng của d qua BC,AC,AB. Các đường thẳng đó đồng quy tại một điểm nằm trên đường tròn ngoại tiếp tam giác ABC(điểm anti steiner của d). Và d được gọi là đường thẳng steiner của điểm đó (gọi là G).

45

Page 46: Tong Hop Cac Dinh Li Hinh Hoc Phang Trong Cac Ki ThiHSG

Chứng minh:

Gọi lần lượt là hình chiếu của H qua ba cạnh \Rightarrow ba điểm này thuộc (O)

ngoại tiếp tam giác ABC và lần lượt thuộc

(mod )Vậy nếu gọi giao điểm của d_a,d_b là G thì G thuộc đường tròn ngoại tiếp tam giác ABC. Tương tự ta có đpcm

Theo hình của bài đường thẳng steiner ta dễ thấy đối xứng với , đối xứng với

Vậy ta có d đúng là đường thẳng steiner của G.

Ta có một tính chất khác của điểm Anti Steiner như sau:Định lí 2:Gọi P là một điểm thuộc đường thẳng d. lần lượt là điểm đối xứng với P qua các

cạnh của tam giác ABC. Ta có các đường tròn cùng đi qua điểm G

Chứng minh :Dễ thấy

(mod )Lại có theo chứng minh trên có:

46

Page 47: Tong Hop Cac Dinh Li Hinh Hoc Phang Trong Cac Ki ThiHSG

(mod )

Suy ra G thuộc . Tương tự có đpcm

I.36)Định lí Napoleon

Định lí:Dựng ra phía ngoài tam giác ABC các tam giác đều BMC,CNA,APB và gọi D,E,F lần lượt là tâm của ba tam giác ấy. Khi đó ta có tam giác DEF đều.

Chứng minh:

Bài này có nhiều cách giải,nếu thuận lợi mình sẽ giới thiệu ,tuy nhiên ở đây mình sẽ trình bày một chứng minh ngắn gọn dựa trên phép quay vecto như sau:

Từ đó có điều cần chứng minh.

Napoleon's Theorem, Two Simple Proofs

On each side of a triangle, erect an equilateral triangle, lying exterior

to the original triangle. Then the segments connecting the centroids of

the three equilateral triangles themselves form an equilateral triangle.

By Dr. Scott Brodie, M.D., Ph.D.

Mount Sinai School of Medicine, NY

47

Page 48: Tong Hop Cac Dinh Li Hinh Hoc Phang Trong Cac Ki ThiHSG

Proof #1 ("Hammer and Tongs"

trigonometry)

In the following, we use standard notations: in the ABC, A

denotes both the vertex A and the correposnding A, a is both BC

and its length. In addition, let G denote the centroid of the

equilateral triangle on side AB, and I denote the centroid of the

equilateral triangle on side AC, etc. Let s denote the length of

segment GI, t the length of segment AG, and u the length of

segment AI. (Geometer's SketchPad illustration.)

Since IAC =  GAB = 30o, we may apply the Law of Cosines to

compute

s2 = u2 + t2 - 2ut·cos(A + 60o). (1)

Since the centroid of a triangle lies along each median, 2/3 of the distance from the vertex to

the midpoint of the opposite side, we have

t = (2/3)·sqrt(3)/2 · c = c/sqrt(3)

u = (2/3)·sqrt(3)/2 · b = b/sqrt(3),

and (1) becomes

3·s2 = b2 + c2 - 2bc·cos(A + 60o). (2)

Expanding(*) the cosine of the sum, and recalling that cos(60o) = 1/2; sin(60o) = sqrt(3)/2, we

have

cos(A + 60o) = cos(A)/2 - sin(A)·sqrt(3)/2. (3)

Substituting (3) into (2) yields

3·s2 = b2 + c2 - bc·cos(A) + sqrt(3)·bc·sin(A). (4)

48

Page 49: Tong Hop Cac Dinh Li Hinh Hoc Phang Trong Cac Ki ThiHSG

Now apply the Law of Cosines to ABC:

a2 = b2 + c2 - 2bc·cos(A). (5)

and recall (as in the derivation of the Law of Sines):

2·Area( ABC) =  bc·sin(A). (6)

Substituting (5) and (6) into (4) gives

3·s2 = (1/2)(a2 + b2 + c2) + 2·sqrt(3)· Area( ABC). (7)

Since (7) is symmetrical in a, b, and c, it follows that the triangle connecting the three centroids

is equilateral, QED.

(*)

Michael Lambrou has suggested a different way to proceed after obtaining (2).

Apply the Law of Cosines to triangles ABE and BCE to express the side BE in two different ways:

b2 + c2 - 2bc·cos(A + 60o) = a2 + b2 - 2ab·cos(C + 60o)

By (2), the left hand side equals 3 times GI. Similarly, the right hand side equals 3 times IH,

wherefrom GI = IH. Considering triangles ACD and ABD instead, we also obtain IH = HG, which

furnishes the proof.

As a bonus, we get AD = BE = CF.

49

Page 50: Tong Hop Cac Dinh Li Hinh Hoc Phang Trong Cac Ki ThiHSG

Proof #2 (an argument by

symmetrization)

Notations are the same as before: let ABC be the

original triangle. Choose D, E, and F exterior to

ABC so that ADB, BEC, and AFC are equilateral

triangles, with centroids G, H, and I, respectively.

(Geometer's SketchPad illustration.) We proceed to

show that HIG = 60o.

Fix I as a center of rotation, and rotate the entire

figure by 120o, and superimpose the rotated copy

on the original figure. Under the rotation, the

CAF maps to itself (C maps to A, A to F, F to C,

while I maps into itself.) Denote by BB, DD, EE, GG,

and HH the images of points B, D, E, G, and H,

respectively.

Connect D to EE and G to HH. By the rigidity of the

rotation, GHI =  GG.HH.I. In particular,

GH = GG.HH. (Geometer's SketchPad illustration.)

Now consider the six triangles that converge on point A. Three of them (ABD, ACF, and A.EE.BB)

are equilateral. Recollect that the angles of a triangle sum to 180o, while the angles around a

point sum to 360o. Since BB.A.F is a copy of BCA, it follows that D.A.EE =  ABC. Finally,

D.A.EE =  ABC, and the pentagon A.BB.EE.D.B is congruent to the pentagon BECAD. It

follows that G.HH = GH. And thus G.HH = GH = GG.HH.

50

Page 51: Tong Hop Cac Dinh Li Hinh Hoc Phang Trong Cac Ki ThiHSG

Repeating the rotation by 120o once more,

and connecting the tips of the equilateral

triangles as above, we obtain the figure on

the right. (Geometer's SketchPad

illustration.) Arguing as above, it is clear that

the central hexagon is equilateral, and that

the six triangles which meet at the center of

rotation are congruent. Therefore, 6·

HIG = 360o, and HIG = 60o. Since (among

the points G,H,I) the choice of the centroid I

was arbitrary, we have shown that GHI is

equiangular, hence equilateral, QED.

I.37)Định lí Morley

Định lí:Trong tam giác ABC. D,E,F lần lượt là giao điểm của các đường chia ba góc trong và cùng kề các cạnh tam giác ABC. Khi đó ta có tam giác DEF đều và được gọi là tam giác Morley.

Chứng minh:

Để ngắn gọn ta đặt và tương tự với các góc kia Như hình vẽ kẻ các

51

Page 52: Tong Hop Cac Dinh Li Hinh Hoc Phang Trong Cac Ki ThiHSG

đường chia trong ở B và C và lần lượt cắt tại D,I. Dễ thấy ID là phân giác của góc . Tại D

dựng góc sao cho Di là phân giác của góc DEF và E thuộc CI và F thuộc BI DEF đều.Lấy và lần lượt là điểm dối xứng với D qua CI và BI và dễ dàng c/m được là hình thang cân với Vì định lí Morley chỉ có một trường hợp nên em xin phép chỉ sử dụng góc thường cho nó đơn giản:

Ta lại có là hình thang cân và trong đường tròn ngoại tiếp

thì sđ A thuộc đường tròn . Từ đó ta có đpcm

Định lý Morley có thể mở rộng các đường chia trong thành các đường chia ngoài, và có thể là giao của đường chia trong với đường chia ngoài(mỗi trường hợp này lại cho ta một tam giác Morley khác nhau và theo thống kê có 36 tam giác Morley như vậy). Sau đó bài toán còn được phát triển và tương ứng được đặt thêm nhiều định nghĩa mới như "góc lửng", "tam giác ngoại lai", "tập hợp đẳng cấu", ...Sau đây là bài toán mở rộng nhất định lý Morley:Nếu chia n (n nguyên dương, n 3) tất cả các góc của một đa giác m cạnh, thì tất cả các giao

của các đường thẳng là các đỉnh phân biệt của một hệ đa giác n cạnh đều, có

thể phân chia làm họ, mỗi họ có đa giác có tâm thẳng hàng.Cách chứng minh và các khái niệm liên quan xin xem thêm tại sách "Lãng mạn toán học" tác giả Hoàng Quý nhà xuất bản giáo dục(Ai có ebook của quyển này up lên thì tốt quá.

I.38)Định lí con bướm với đường tròn

Định lí:Cho đường tròn (O) và dây cung AB. I là trung điểm của AB. Qua I vẽ hai dây cung tùy ý MN và PQ sao cho MP và NQ cắt AB tại E,F. Khi đó I là trung điểm của EF.

Chứng minh:

Gọi K,T là trung điểm MP và NQ. Nên OIEK, OIFT là tứ giác nội tiếp

52

Page 53: Tong Hop Cac Dinh Li Hinh Hoc Phang Trong Cac Ki ThiHSG

(mod ) (mod )

Ta lại có (mod ) cân tại O I là trung điểm EF

I.39)Định lí con bướm với cặp đường thẳngĐịnh lí:Cho tam giác . Lấy là trung điểm của . Qua kẻ các đường thẳng cắt

tại , đường thẳng cắt tại . Gọi cắt tại . Khi đó ta có là trung điểm cưa Chứng minh:

Áp dụng định lí menelaus trong tam giác ta có các hệ thức sau:

(1)

(2)từ (1) và (2) ta có:

Vậy là trung điểm của . (ĐPCM)  

I.6) Định lý Desargues

Định lý: Cho tam giác ABC và tam giác A'B'C'. Khi đó AA', BB', CC' đồng quy khi và chỉ khi các giao điểm của BC và B'C', CA và C'A', AB và A'B' thẳng hàng.

53

Page 54: Tong Hop Cac Dinh Li Hinh Hoc Phang Trong Cac Ki ThiHSG

Chứng minh:

Gọi X, Y, Z là lần lượt là các giao điểm của các cặp cạnh BC và B’C’, CA và C’A’, AB và A’B’ .

Phần thuận:Giả sử các đường thẳng AA’, BB’, CC’ đồng quy tại S. Ta chứng minh X, Y, Z thẳng hàng.Áp dụng định lí Menelaus cho tam giác SBC với cát tuyến XB'C' ta có:

hay

Tương tự, ta có:

Nhân từng vế các đẳng thức trên lại với nhau, và theo định lí Menelaus suy ra X, Y, Z thẳng hàng.

Phần đảo:Giả sử các điểm X, Y, Z thẳng hàng. Ta chứng minh các đường thẳng AA’, BB’, CC’ đồng quy.Gọi S là giao điểm của AA’ và BB’. SC cắt đường thẳng AC’ tại C”.Xét 2 tam giác ABC và A’B’C” có các đường nối các đỉnh tương ứng đồng quy, do đó theo phần thuận giao điểm của các cạnh tương ứng cũng đồng quy.Ta thấy AB cắt A’B’ tại Z, AC cắt A’C” tại Y (do A’, C’, C” thẳng hàng), suy ra giao điểm X’ của BC và B’C” phải thuộc YZ. Tức là X’ là giao của YZ và BC nên X’ trùng với X.Suy ra C” trùng với C’, hay AA’, BB’, CC’ đồng quy.

I.40 Định Lí BlaikieĐịnh lí: Cho tam giác ABC và đường thẳng d sao cho d cắt BC,CA,AB lần lượt ở M,N,P. Gọi S là 1

54

Page 55: Tong Hop Cac Dinh Li Hinh Hoc Phang Trong Cac Ki ThiHSG

điểm bất kì trên d. Gọi M',N',P' lần lượt là điểm đối xứng của M,N,P qua S. Khi đó AM',BN',CP' đồng quy tại một điểm P và ta gọi P là điểm Blaikie của d và S đối với tam giác ABC.

Chứng Minh : Có thể cho nằm giữa .Giả sử cắt tại . Ta chứng mình thẳng hàng .Xét tam giác với điểm . Ta cần cm :

Xét tam giác với điểm thẳng hàng trên cạnh :

(1)

Xét tam giác với điểm thẳng hàng trên cạnh :

(2)

Nhân vế (1),(2) và rút gọn , chú ý ta được :

Chú ý là và nên ta có đpcm.

I.41)Định lí chùm đường thẳng đồng quy.

Định lí 1: Ba đường thẳng đồng quy thì định ra trên hai đường thẳng song song những đoạn thẳng tỉ lệ.

Chứng minh:  

55

Page 56: Tong Hop Cac Dinh Li Hinh Hoc Phang Trong Cac Ki ThiHSG

I.42)Đường tròn Apollonius

Định lí:Cho hai điểm A và B cố định. Khi đó quĩ tích điểm M sao cho là một đường tròn cố định được gọi là đường tròn Apollonius.

Chứng minh:

Lấy D,E thuộc đường thẳng AB sao cho

a\Phần thuận

Ta có ME là phân giác của M thuộc đường tròn đường kính DE.b\Phần đảo

Lấy M' thuộc đường tròn đường kính DE .

Lại có ME là phân giác của đpcm

I.43)Định lí Blanchet

Định lí:Cho tam giác ABC có AH là đường cao ứng với cạnh BC. Gọi I là một diểm tùy ý thuộc đoạn AH.các đoạn thẳng BI,CI cắt các cạnh tam giác tại E và F.Chứng minh rằng HA là phân giác của góc EHF

56

Page 57: Tong Hop Cac Dinh Li Hinh Hoc Phang Trong Cac Ki ThiHSG

Chứng minh:Qua I kẻ đường thẳng song song với BC cắt AB,HF,HE,AC lần lượt tại M,N,P,Q.Ta chỉ cần chứng minh IN=IP là xong !!Theo Thales:

Do đó :

Hiển nhiên Nên IN =IP ---------------> dpcm

I.44)Mở rộng của định lí BlanchetĐịnh lí:Cho tam giác ABC, lấy lần lượt thuộc các đoạn sao cho 3 đường thẳng đồng quy tại một điểm.Gọi là giao điểm của và .Gọi H là hình

chiếu của xuống . Chứng minh rằng là phân giác của Chứng minh:

57

Page 58: Tong Hop Cac Dinh Li Hinh Hoc Phang Trong Cac Ki ThiHSG

Sử dụng Ceva và Menelaus ta chứng minh được

Theo định lí về chùm điều hòa ta lại có

------>đpcm (hệ quả của chùm điều hòa)

I.45) Định lí Jacobi:

Định lí:

Cho tam giác ABC và các điểm trên mặt phẳng sao cho: ,

, .Khi đó đồng quy tại điểm Jacobi N.

Chứng minh:

Do đồng quy tại và tương tự cho nên áp dụng định lý Ceva dạng Sin

58

Page 59: Tong Hop Cac Dinh Li Hinh Hoc Phang Trong Cac Ki ThiHSG

ta có:

->Xây dựng hai đẳng thức tương tự cho rồi nhân 3 đẳng thức trên với nhau ta được:

Như vậy đồng quy theo định lý Ceva dạng Sin.

I.46)Định lí Kiepert

Định lí:Dựng ra phía ngoài tam giác ABC các tam giác cân đồng dạngBCM,CAN,ABP(Cân ở M,N,P).Khi ấy ta có AM,BN,CP đồng quy

Chứng minh

Do các tam giác BCM,CAN,ABP cân và đồng dạng nên dễ thấy:

Theo định lí Jacobi ta có điều cần chứng minh.  

I.47)Định lí Kariya

Định lí :Cho tam giác ABC nhận (I) là đường tròn nội tiếp.Ở phía ngoài tam giác lấy các điểm M,N,P sao

59

Page 60: Tong Hop Cac Dinh Li Hinh Hoc Phang Trong Cac Ki ThiHSG

cho IM =IN=IP và IM,IN,IP tương ứng vuông góc BC,CA,AB.Khi đó ta có AM,BN,CP đồng quy.Chứng minh:

Từ giả thiết bài toán dễ suy ra:(c.g.c)

Do đó:

=>Tương tự:

Theo định lí Jacobi ta có điều cần chứng minh.

I.48)Cực trực giao

Đây là một khái niệm mở rộng kết quả về trực tâm tam giác.

Định lí:Cho tam giác ABC. d là một đường thẳng bất kì trong mặt phẳng. Gọi lần lượt là hình chiếu của A,B,C trên d. Gọi lần lượ là hình chiếu của trên BC,CA,AB. Khi đó

đồng quy tại một điểm gọi là cực trực giao của đường thẳng d đối với ABC.

60

Page 61: Tong Hop Cac Dinh Li Hinh Hoc Phang Trong Cac Ki ThiHSG

Chứng minh:Áp dụng định lí carnot ta có đpcm

(hiển nhiên đúng)Trực tâm là trường hợp khi d trùng với một trong ba cạnh của ABC

(Xem them FG200406.bdf)

I.49)Khái niệm tam giác hình chiếu ,công thức Euler về diện tích tam giác hình chiếu

Định lí:

Cho là đường tròn nội tiếp tam giác ABC.Cho điểm M nằm trong tam giác.Gọi là hình chiếu của M lên ba cạnh BC,AC,AB. Khi đó ta gọi là tam giác hình chiếu của điểm M đối với tam giác ABC. Ta có công thức Euler về diện tích của tam giác hình chiếu:

61

Page 62: Tong Hop Cac Dinh Li Hinh Hoc Phang Trong Cac Ki ThiHSG

Chứng minh:(Mashimaru)

Ta có tứ giác nội tiếp đường tròn đường kính nên , tương tự

thì .Gọi là giao điểm của với đường tròn ngoại tiếp tam giác . Ta có:

và .Mặt khác:

Xét tam giác , theo định lý sin ta có:

Suy ra: . Từ đó ta có:

Mặt khác, ta lại có: và nên:

(Xem them pedal(2).bdf)

I.50)Khái niệm hai điểm đẳng giác

Định lí:Cho tam giác ABC. M là một điểm nằm trong tam giác.1/Khi đó các đường thẳng đối xứng với AM,BM,CM qua tia phân giác đồng quy tại M'. M' được gọi là điểm đẳng giác của M.2/Lần lượt đặt D,E,F và D',E',F' là chân các đường cao hạ từ M và M' xuông BC,AC,AB. a/Khi đó D,E,F,D'E',F' cùng thuộc một đường tròn tâm O. Và O là trung điểm của M và M'.

b/Khi đó cũng có .

62

Page 63: Tong Hop Cac Dinh Li Hinh Hoc Phang Trong Cac Ki ThiHSG

Chứng minh:1/(hình 1)

Tương tự áp dụng định lí Ceva thuận và đảo ta có đpcm12/a/(hình 2)

(mod )

(mod )

(mod )

63

Page 64: Tong Hop Cac Dinh Li Hinh Hoc Phang Trong Cac Ki ThiHSG

(mod )

(mod )

nội tiếp. Trung trực FF' và DD' gặp nhau tại trung điểm O của MM'(t/c đường trung bình hình thang) F,F'D,D' thuộc đường tròn tâm O. Tương tự ta có đpcm.b/(hình 3)

lại có (g.g) Tương tự ta có đpcm

Trường hợp M là trọng tâm thì M' là điểm đối trung (AM,BM,CM lần lượt được gọi là các đường đối

trung) có tính chất rằng: min  

I.51)Khái niệm tứ giác toàn phần .

Khái niệm:Một tứ giác toàn phần là một hình được tạo nên bởi bốn đường thẳng, từng đôi một cắt nhau nhưng không có ba đường nào đồng qui. Một hình tứ giác toàn phần có 4 cạnh là 4 đường thẳng ấy, có 6 đỉnh là 6 giao điểm của chúng và 3 đường chéo là 3 đoạn đi qua đỉnh đối diện (chú ý hai đỉnh này không cùng thuộc một cạnh).

Chúng ta có một kết quả cơ bản và thú vị về tứ giác này như sau:

Định lí :Trong hình tứ giác toàn phần cặp đỉnh đối diện nằm trên một đường chéo và cặp giao điểm của đường chéo đó với hai đường chéo còn lại lập thành một hàng điểm điều hòa.

64

Page 65: Tong Hop Cac Dinh Li Hinh Hoc Phang Trong Cac Ki ThiHSG

Xem hình vẽ sau:

 

I.52)Đường thẳng Droz-Farny

Định lí:Cho hai đường thẳng bất kì vuông góc với nhau tại trực tâm của tam giác ABC. Chúng tương ứng cắt các cạnh BC, AC, AB tại X, X'; Y, Y'; Z, Z'. Khi đó ta có tương ứng là các trung

điểm của thẳng hàng

Chứng minh:(dịch từ bài viết của Jean-Louis Ayme)

65

Page 66: Tong Hop Cac Dinh Li Hinh Hoc Phang Trong Cac Ki ThiHSG

Đặt C là đường tròn ngoại tiếp ABC. là đường tròn ngoại tiếp HXX' và là điểm đối xứng với H qua BC. Tương tự với các đường tròn khác.

có tâm lần lượt là .XX' là đường kính của đường tròn nằm

trên đường tròn này là giao của C và và .Áp dụng định lí Collings(trong bài viết điểm Anti-steiner) với đường thẳng XYZ đi qua H, ta có

đồng quy tại N trên C. Áp dụng định lí Miquel cho tam giác XNY với các điểm cùng đi qua M. Tương tự cũng đi qua M.

Như vậy cùng đi qua H và M (đồng trục). Nên tâm của chúng thẳng hàng.Các tài liệu có liên quan:

(Xem them FG200426.bdf; FG200427.bdf)

I.53)Đường tròn Droz-Farny

Định lí:Cho điểm P bất kì và tam giác ABC. Điểm Q là điểm đẳng giác với P đối với tam giác ABC. Chân các đường vuông góc với các cạnh BC,AC,AB của P là . Lấy làm tâm vẽ đường tròn đi qua Q cắt BC tại . định nghĩa tương tự. Khi đó cùng thuộc đường tròn tâm P.

Chứng minh:O là trung điểm của PQ

Ta đã biết O cách đều Mà :

(công thức đường trung tuyến)Hoàn toàn tương tự ta sẽ có các điểm cách đều P(điều cần chứng minh).  

I.54)Định lí Van Aubel về tứ giác và các hình vuông dựng trên cạnh.

66

Page 67: Tong Hop Cac Dinh Li Hinh Hoc Phang Trong Cac Ki ThiHSG

Định lí:Về phía ngoài tứ giác ABCD ta dựng các hình vuông ABUI,BCQP,CDJW,DAFE với các tâm tương ứng là T,N,V,M.Khi đó ta có TV và MN vuông góc với nhau.

Chứng minh:

Gọi O là trung điểm đoạn AC.

Ta thấy:

Do vậy:

Chú ý F là phép đồng dạng góc với tỉ số 1 có O là điểm bất động tức là F là phép quay tâm O góc . (1)Như vậy : (2)Tương tự ta có: (3)

Từ (1) ,(2) và (3) ta có điều cần chứng minh.

I.2 Mở rộng định lí Menelaus theo diện tích.Định lí:Cho tam giác ABC và 3 điểm M,N,P lần lượt nằm trên BC,CA,AB.Khi đó ta có:

67

Page 68: Tong Hop Cac Dinh Li Hinh Hoc Phang Trong Cac Ki ThiHSG

Chứng minhGọi là vector chỉ phương của Ta có:

mặt khác :

tương tự:

Ta suy ra:

I.55)Hệ thức Van Aubel

Định lí:Cho tam giác ABC và các điểm D,E,F lần lượt thuộc BC,CA,AB sao cho AD,BE,CF đồng quy ở S.Khi đó ta có:

Và 2 hệ thức tương tự.Chứng minh:

68

Page 69: Tong Hop Cac Dinh Li Hinh Hoc Phang Trong Cac Ki ThiHSG

Cách chứng minh dưới đây rất độc đáo mình đọc trong một file của anh Darij Grinberg(xem cuối bài)Chúng ta sử dụng định lí Gergonne -Euler(mục I.28)cho điểm A với tam giác SBC.Ta có:

=>

=>  

(Xem them GergonneEuler.bdf)

I.56)Định lí Pithot

Định lí: Tứ giác lồi ABCD là tứ giác ngoại tiếp khi và chỉ khi :AB+CD =BC+DA

Chứng minh:

+)Chiều thuận: ABCD là tứ giác ngoại tiếp thì: AB+CD=BC+DA

Phần này chứng minh hoàn toàn đơn giản xin dành cho bạn đọc.

+)Chiều đảo:Nếu tứ giác lồi ABCD thỏa mãn : AB+CD=BC+DA (*)thì nó là tứ giác ngoại tiếp.

69

Page 70: Tong Hop Cac Dinh Li Hinh Hoc Phang Trong Cac Ki ThiHSG

Không giảm tổng quát giả sử : ,khi ấy từ (*) cũng suy ra: .Khi ấy trên AD,DC tương ứng tồn tại hai điểm N,M sao cho:AN=AB,CM=CB.Từ (*) sẽ có: DN=DM.

Do đó các đường phân giác của các góc tại đỉnh A,D,C sẽ là ba đường trung trực của tam giác BMN nên chúng đồng quy tại một điểm (mà ta kí hiệu là O).Nhận thấy O cách đều 4 cạnh tứ giác nên ta có điều cần chứng minh.

I.57)Định lí Johnson

Định lí:Cho ba đường tròn có cùng bán kính R với tâm lần lượt là M,N,P và cùng đi qua một điểm A.Khi ấy ba giao điểm khác A của ba đường tròn ấy cùng nằm trên một đường tròn có bán kính là R.

Chứng minh:

Mình gặp định lí này và không hề biết chứng minh của nó ,ở đây mình trình bày một phép chứng

70

Page 71: Tong Hop Cac Dinh Li Hinh Hoc Phang Trong Cac Ki ThiHSG

minh mà mình nghĩ ra như sau mà theo mình nó là một chứng minh dài.Chúng ta kí hiệu các giao điểm khác A là B,C,D như hình vẽ và gọi Q là tâm (BCD).Ta thấy : PA=PB=MA=MB nên MAPB là hình thoi =>M là điểm đối xứng của P qua AB.Chú ý bán kính của (P) và (M) là bằng nhau nên suy ra (P) và (M) đối xứng với nhau qua AB.(1)Tương tự (P) và (N) đối xứng với nhau qua AC (2).(P) là đường tròn ngoại tiếp tam giác ABC (3)

Từ (1) ,(2) và (3) ta sẽ suy ra D là trực tâm tam giác ABC.Đến đây công việc còn lại đã rất đơn giản........................

I.58)Định lí Eyeball

Định lí: Cho hai đường tròn(O) và (O') ngoài nhau.Hai tiếp tuyến kẻ từ O tới (O') cắt (O') tại C,D.Hai tiếp tuyến kẻ từ O' tới (O) cắt (O) tại A,B.Khi đó ta có : AB =CD.

Chứng minh:

Chúng ta kí hiệu các điểm A,B,C,D như hình vẽ.OM,O'N tương ứng là các tiếp tuyến của (O') và (O).AD cắt lại (O') ở P.

Ta thấy:

(1)

(2)

Cộng theo vế (1) và (2) ta có:

Do đó CP là đường kính của (O').

Vì vậy sẽ có: Hoàn toàn tương tự ta cũng có các đẳng thức góc khác nên sẽ có ABCD là hình chữ nhật => AB=CD.  

I.59)Bổ đề Haruki

Bổ đề:Cho AB và CD là hai dây cung không cắt nhau của cùng một đường tròn và P là một điểm bất kì trên cung AB không chứa CD của đường tròn ấy.Gọi E và F lần lượt là giao điểm của PC,PD với AB.Thế thì giá trị biểu thức sau là không đổi:

Chứng minh:

71

Page 72: Tong Hop Cac Dinh Li Hinh Hoc Phang Trong Cac Ki ThiHSG

(AED) cắt lại AB ở G.

Ta thấy: (Không đổi) => G cố định => BG không đổi (1)

Mặt khác :

(2)

Từ (1) và (2 ) suy ra dpcm.

(Xem them FG200809.bdf)

I.60)Bài toán Langley

Bài toán:Cho cân tại A có . Trên cạnh lấy điểm sao cho

, .Tính ?

Lời giải:

72

Page 73: Tong Hop Cac Dinh Li Hinh Hoc Phang Trong Cac Ki ThiHSG

Đặt Trên cạnh lấy điểm sao cho .Khi đó cân tại

Mặt khác cân tại (do )

Suy ra

Do đó đều( )

Lại có cân tại

Ta suy ra cân tại

I.61)Định lí Paul Yiu về đường tròn bàng tiếpĐịnh lí:Cho các đường tròn bàng tiếp góc tiếp xúc với 3 cạnh lần lượt tại

.Các đường thẳng qua giao nhau tại .Các đường thẳng qua giao nhau tại .Chứng minh rằng các bộ ba điểm

thẳng hàng và các đường thẳng qua chúng đồng quy.Chứng minh:

73

Page 74: Tong Hop Cac Dinh Li Hinh Hoc Phang Trong Cac Ki ThiHSG

Gọi là giao điểm của và với

Áp dụng định lí Menelaus cho bộ 3 điểm tương ứng với và bộ 3 điểm

tương ứng với ta được:

Nhân (1) và (2) cho ta:

Mặt khác

Áp dụng định lí Menelaus cho với bộ 3 điểm và ta được:

Nhân (1) với (2) ta được:

Ta suy ra:

Vậy 3 điểm thẳng hàng

74

Page 75: Tong Hop Cac Dinh Li Hinh Hoc Phang Trong Cac Ki ThiHSG

Các bộ 3 điểm còn lại chứng minh tương tự.

Chứng minh 3 đường thẳng đồng quyGọi là lần lượt là giao điểm của với Như chứng minh ở trên ta có:

tương tự với thì:

Ta suy ra:

Do đó thẳng hàng

Mặt khác theo định lí Desagues thì các đường thẳng đồng quy.Ta suy ra dpcm

I.62)Định lí Maxwell

Định lí: Cho ABC và một điểm P, các cạnh của A'B'C' song song với các đường thẳng đi qua một đỉnh ABC và điểm P. Qua A',B',C' kẻ các đường thẳng song song với các cạnh của ABC. Khi đó ta có các đường thẳng này đồng quy tại một điểm P'.

Chứng minh:

Dễ dàng c/m được các góc và . Tương tự áp dụng định lí Ceva Sin ta có đpcm.

Projective Proof of Maxwell's Theorem

75

Page 76: Tong Hop Cac Dinh Li Hinh Hoc Phang Trong Cac Ki ThiHSG

Maxwell's theorem links the sides and cevians in one triangle to the cevians and side lines in

another:

 

Given two triangles ABC and MNP. Assume that the cevians in ΔABC parallel to the

sides of ΔMNP are concurrent. Then the cevians in ΔMNP parallel to the sides of

ΔABC are also concurrent.

Michel Cabart came up with a generalization that shows that Maxwell's theorem is of projective

nature. The main tool is the projective reformulation of Ceva's theorem.

By Michel Cabart

9 May, 2008

Recollect the projective formulation of Ceva's theorem: Given an arbitrary line L, the cevians in

ΔABC concur iff

(1)R = (CBA'A0)·(ACB'B0)·(BAC'C0)

= -1

where (WXYZ) denotes the cross-ratio YW/YX : ZW/ZX and the points A', B', C', A0, B0, C0 are

defined as in the diagram below, where L is an arbitrary straight line.

76

Page 77: Tong Hop Cac Dinh Li Hinh Hoc Phang Trong Cac Ki ThiHSG

 

In addition, let A1, B1, C1 be the intersection with L of the cevians AA', BB', and CC'.

Projection from A to the line L send the first quadruple in (1) onto B0, C0, A1, A0:

 (CBA'A0) =

A(B0C0A1A0)

Similarly,

 

(ACB'B0) =

B(C0A0B1B0),

(BAC'C0) =

C(A0B0C1C0)

Thus the condition (1) can be written (cross-ratios being unchanged):

(2) R = (A0B0C1C0)·(B0C0A1A0)·(C0A0B1B0)

77

Page 78: Tong Hop Cac Dinh Li Hinh Hoc Phang Trong Cac Ki ThiHSG

= -1

Proof of Maxwell's theorem

 

Assume there are two triangles ABC and MNP and two triples of points on L such that A0, B0, C0

are intersections of L with sides of ΔABC and also with cevians of ΔMNP; A1, B1, C1 are

intersections of L with sides of ΔMNP and also with cevians of ΔABC.

For ΔABC, we calculate

 R0 =

(A0B0C1C0)·(B0C0A1A0)·(C0A0B1B0).

For ΔMNP, we calculate

  R1 = (A1B1C0C1).(B1C1A0A1).

78

Page 79: Tong Hop Cac Dinh Li Hinh Hoc Phang Trong Cac Ki ThiHSG

(A1B1C0C1)

We have the identity R0·R1 = 1 (as can be seen by developing cross-ratios). Hence R0 = -1 iff R1

= -1, or: the cevians in ΔABC concur iff the cevians in ΔMNP concur. Choosing the infinite line

as line L gives Maxwell's theorem as a particular case. The case where sides and cevians in

ΔMNP are taken perpendicular (or in fact any angle) to ΔABC is straightforward via an

homography (in projective terms) or a rotation (in Euclidean terms).

Note: a dynamic illustration is available on a separate page.

Maxwell Theorem via the Center of Gravity

Michel Cabart

30 April, 2008

Below we offer a proof of Maxwell's theorem that is based on the notion of barycenter.

Maxwell's theorem states the following fact:

 Given ΔABC and a point G, the sides of ΔMNP are parallel to the cevians in ΔABC

through G. Then the cevians in ΔMNP parallel to the sides of ΔABC are concurrent.

 

Below, the vector joining point A to B will be written in bold, so that, for example, AB = - BA.

79

Page 80: Tong Hop Cac Dinh Li Hinh Hoc Phang Trong Cac Ki ThiHSG

In ΔABC with A', B', C' on sides opposite the vertices A, B, and C, the fact that the cevians AA',

BB', CC' concur in point G is equivalent to either of the two conditions:

 There is a triple of real numbers (a, b, c), unique up to a non-zero factor,

such that

(1) aGA + bGB + cGC = 0

   

 There is a triple of real numbers (a, b, c), unique up to a non-zero factor,

such that

(2)

A' = Z(B, b; C, c)

B' = Z(A, a; C, c)

C' = Z(A, a; B, b),

where Z(X, x; Y, y) denotes the barycenter of two material points X and Y with masses x at X

and y at Y.

Let's suppose lines AA', BB' and CC' intersect.

Step 1: NP, PM, MN are parallel to GA, GB, GC means there exists x, y, z such that NP = xGA,

PM = yGB, MN = zGC. As NP + PM + MN = 0, xGA + yGB + zGC = 0. A comparison with (1)

shows that (x, y, z) is a multiple of (a, b, c). We can assume (x, y, z) = (a, b, c). Thus NP = aGA,

PM = bGB, MN = cGC.

Step 2: MM', NN', PP' are parallel to BC, AC, AB meaning there is (m, n, p) such that MM' =

mBC, NN' = nAC, PP' = pAB. The first equality yields

 MM' = m(GC - GB) = (m/c)MN +

(m/b)MP

so that

  M' = Z(N, 1/c; P, 1/b) = Z(N, b;

80

Page 81: Tong Hop Cac Dinh Li Hinh Hoc Phang Trong Cac Ki ThiHSG

C, c)

by multiplying by bc. Similarly,

 

N' = Z(M, a; P, c)

and

P' = Z(M, a; N, b).

This proves the theorem thanks to (2).

Maxwell's Theorem

The applet suggests the following theorem [Prasolov, 11.48, Pedoe, 6.1, 8.3, 28.4]:

 

Given ΔABC and a point P, the sides of ΔA'B'C' are parallel to the cevians in ΔABC

through P. Prove that the cevians in ΔA'B'C' parallel to the sides of ΔABC are

concurrent.

A similar statement is true if the lines are taken to be perpendicular, instead of parallel.

Furthermore, the two are clearly equivalent. We'll prove the latter.

Triangles A'B'C' with sides parallel to the given set of cevians are all similar. It therefore

sufficient to establish the theorem for any one of those triangles. Consider the circumcenters

OA, OB, and OC, of triangles PBC, PAC, PAB, respectively. The circumcircles of triangles PBC and

PAC share chord PC, so that OAOB is perpendicular to PC. Similarly, OAOC PB and OBOC PA,

which means that ΔOAOBOC is one of the family A'B'C'. The lines through its vertices

perpendicular to the sides of ΔABC are exactly the perpendicular bisectors of the latter, which

are known to meet at a point.

Two triangles ABC and A'B'C' are said to be orthologic if perpendiculars from A, B, C to B'C',

A'C', A'B' are concurrent. The point of concurrency is known as the orthologic center of ΔABC

with respect to ΔA'B'C'. Maxwell's theorem justifies the symmetry of the definition: if the

perpendiculars from the vertices of one of the triangle to the sides of the other are concurrent,

then the perpendiculars from the vertices of the latter to the sides of the former are also

81

Page 82: Tong Hop Cac Dinh Li Hinh Hoc Phang Trong Cac Ki ThiHSG

concurrent. As an example, in any triangle, the associated medial and contact triangles are

orthologic.

We can use complex variables and the real product [Andreescu, 4.1] of complex numbers to

easily establish the perpendicular case of Maxwell's theorem. For two complex numbers u and v

define the real product as

 u·v = (uv* +

vu*)/2,

where the asterisk denotes the complex conjugate. Assuming u = u1 + iu2 and v = v1 + iv2,

 u·v = u1v1 +

u2v2.

In other words, the real product of two complex numbers is exactly the scalar product of the

2D-vectors represented by these complex numbers. It follows that for four complex numbers u,

U, v, V the lines joining them pairwise are perpendicular iff

 (U - u)·(V - v) =

0.

Assume now the complex coordinates of the vertices of triangles ABC and A'B'C' are a, b, c and

a', b', c'. The perpendicular from A to B'C' has the equation

(1)(z - a)·(c' - b') =

0.

Similarly the perpendiculars from B and C are given by the equations

(2)

(z - b)·(a' - c') = 0

and

(z - c)·(b' - a') = 0.

Adding the three up eliminates z:

82

Page 83: Tong Hop Cac Dinh Li Hinh Hoc Phang Trong Cac Ki ThiHSG

(3)a·(c' - b') + b·(a' - c') + c·(b' - a')

= 0,

which is the condition for the perpendiculars from A, B, C to B'C', A'C', A'B' to be concurrent.

Indeed, if (3) holds then, for any z,

(4)(z - a)·(c' - b') + (z - b)·(a' - c') + (z - c)·(b' -

a') = 0.

Now let's see if we can choose z the right way. First, for any z on the perpendicular from a to

b'c', the first term in (4) vanishes. The choice of z as the intersection of the perpendiculars from

a and b, eliminates the first two terms, which makes the third term also 0.

Importantly, (3) can be regrouped into

(3')a'·(c - b) + b'·(a - c) + c'·(b - a)

= 0,

in which the triangles switched the roles. (3') is the condition for the concurrency of the

perpendiculars from A', B', C' onto the sides of triangle ABC. Q.E.D.

As an example, the pedal triangle of any point is orthologic to the base triangle. Also, Gergonne

and medial triangles associated with the given one are orthologic.

Remark 1

The theorem bears name of James Clerk Maxwell, a famous physicist. His proof is different from

anything above yet beautifully simple. He drew 4 pairwise intersecting circles centered at A, B,

C and P. Taken by three, the circles define 4 radical centers. Let A' be the radical center of the

circle centered at B, C, P. B', C', P' are defind cyclically. Then, say, A' and P' lie on the common

chord of the circles B and C, which makes A'P' perpendicular to BC, etc.

Remark 2

83

Page 84: Tong Hop Cac Dinh Li Hinh Hoc Phang Trong Cac Ki ThiHSG

There is another proof of Maxwell's theorem based on the notion of barycenter and another one

of a generalization that shows surprisingly the projective nature of the theorem.

References

1. T. Andreescu, D. Andrica, Complex Numbers From A to ... Z, Birkhäuser, 2006

2. D. Pedoe, Geometry: A Comprehensive Course, Dover, 1970

3. V. V. Prasolov, Problems in Planimetry, v 1, Nauka, Moscow, 1986 (Russian)

I.62)Định lí Maxwell

Định lí: Cho ABC và một điểm P, các cạnh của A'B'C' song song với các đường thẳng đi qua một đỉnh ABC và điểm P. Qua A',B',C' kẻ các đường thẳng song song với các cạnh của ABC. Khi đó ta có các đường thẳng này đồng quy tại một điểm P'.

Chứng minh:

Dễ dàng c/m được các góc và . Tương tự áp dụng định lí Ceva Sin ta có đpcm.

I.63)Định lí Brahmagupta về tứ giác nội tiếp có hai đường chéo vuông góc.

Định lí:Cho tứ giác nội tiếp ABCD có AC vuông góc với BD tại S. Khi đó đoạn nối trung điểm một cạnh với S sẽ vuông góc với cạnh đối diện.Chứng minh:

84

Page 85: Tong Hop Cac Dinh Li Hinh Hoc Phang Trong Cac Ki ThiHSG

Ta chứng minh đại diện,chẳng hạn gọi M là trung điểm BC ta cần chứng minh MS vuông góc với AD.Thật vậy,MS cắt AD ở H.

Ta có: ,M là trung điểm BC nên MS=MC.Do đó:

Dễ =>dpcm

I.64)Định lí Schooten

Định lí: Cho tam giác đều ABC nhận (O) là đường tròn ngoại tiếp.Khi đó với mọi điểm S nằm trên (O) thì một trong 3 đoạn SA,SB,SC có một đoạn có độ dài bằng tổng độ dài hai đoạn còn lại.

Chứng minh:

85

Page 86: Tong Hop Cac Dinh Li Hinh Hoc Phang Trong Cac Ki ThiHSG

Không giảm tổng quát ,giả sử S thuộc cung BC nhỏ.Ta sẽ chứng minh SA=SB+SC.Thật vậy:Gọi I là một điểm trên đoạn SA sao cho SI=SC.Đến đây công việc của chúng ta là chứng minh AI=SB (*) mà điều này thì khá đơn giản ,chỉ cần để ý một chút:Dễ thấy tam giác SIC đều nên (1)

Mà:CA=CB (2) và (3)Từ (1),(2) và (3) ta dễ suy ra (*)

I.65)Định lí Bottema

Định lí:Về phía ngoài tam giác ABC ta dựng hai hình vuông ABDE, ACFG .Gọi M là trung điểm DF.Thế thì Vị trí điểm M không phụ thuộc vào vị trí điểm A và tam giác MBC vuông cân tại M.

Chứng minh:

86

Page 87: Tong Hop Cac Dinh Li Hinh Hoc Phang Trong Cac Ki ThiHSG

Chúng ta sẽ chứng minh ý hai bởi từ điều đó cũng suy ra ngay ý một.Vậy công việc của chúng ta là chứng minh tam giác MBC vuông cân tại M.

Bài này có nhiều cách giải,ở đây ma 29 xin trình bày bằng pp phép biến hình

Kí hiệu: là chỉ phép quay tâm góc quay ,giả sử chiều dương là ngược chiều kim đồng hồ.Ta thấy:

Mà F là phép quay với góc quay nên tâm phép quay này chính là M. Dựa vào cách xác định tâm của tích hai phép quay ta dễ có dpcm

I.66)Định lí Pompeiu

Định lí;Cho tam giác ABC đều ,và một điểm D trên mặt phẳng tam giác.Khi đó luôn tồn tại một tam giác với độ dài các cạnh là DA,DB,DC.

Chứng minh:

Chúng ta sẽ dùng bất đẳng thức Ptolemy (Xem mục I.10)để giải quyết bài này một cách cực kì

nhanh gọn!Bây giờ,theo nguyên lí khởi đầu cực trị trong ba đoạn DA,DB,DC sẽ có một đoạn có độ dài lớn nhất.Không giảm tổng quát ,giả sử đó là DA.Đến đây ta chỉ cần chứng minh ($)là xong.Kí hiệu a là độ dài cạnh tam giác đều ABCSử dụng bất đẳng thức Ptolemy ta có:

87

Page 88: Tong Hop Cac Dinh Li Hinh Hoc Phang Trong Cac Ki ThiHSG

Đến đây thì thấy tiền ($)rồi

I.67)Định lí ZaslavskyĐịnh lí:Cho tam giác và điểm .Tam giác là ảnh của tam giác qua phép đối xứng tâm .Từ kẻ các đường thẳng song song với nhau cắt tại

.Chứng minh rằng thẳng hàng.

Chứng minh:

Từ kẻ đường thẳng song song với cắt tại .

Vì là ảnh của qua phép đối xứng tâm nên là ảnh của qua phép đối xứng tâm

suy ra Ta có:

Ta suy ra dpcm

I.68)Định lí ArchimedesĐịnh lí:Cho là trung điểm , điểm chuyển động tùy ý trên .Từ kẻ .Chứng minh rằng

88

Page 89: Tong Hop Cac Dinh Li Hinh Hoc Phang Trong Cac Ki ThiHSG

Chứng minh:Trên tia dựng điểm sao cho .

Ta có:

đồng thời suy ra Vậy đồng thời là đường cao, đường trung tuyến tam giác nên tam giác cân tại M suy ra do đó là trung điểm cạnh hay nói cách khác (dpcm)

I.69) Định lí UrquhartĐịnh lí:Cho hai bộ ba điểm thẳng hàng và , là giao điểm của và .Chứng minh rằng khi và chỉ khi .

89

Page 90: Tong Hop Cac Dinh Li Hinh Hoc Phang Trong Cac Ki ThiHSG

Chứng minh:

Đầu tiên ta cần chứng minh bổ đề sau:Trong tam giác ABC ta có với p là nửa chu vi và a=BC.

Ta có:

mặt khác

.

nên suy ra

do nên nghịch đảo hai vế ta được dpcm

Trở lại bài toán gọi các góc như trên hình vẽ ta có:

(dpcm)

I.70)Định lí Mairon WaltersĐịnh lí:Cho tam giác ABC và các đường thẳng chia 3 cạnh đối diện như hình vẽ.Chứng minh rằng

90

Page 91: Tong Hop Cac Dinh Li Hinh Hoc Phang Trong Cac Ki ThiHSG

Chứng minh::Trước tiên ta cần chứng minh bổ đề sau:Cho tam giác điểm di động trên đường thẳng

sao cho .Giả sử giao nhau tại thì chia đoạn

theo tỉ số .

Giả sử thì ta có:

lại do cùng phương với nên tồn tại một số sao cho Mặt khác cách biểu diễn này là duy nhất nên ta có đồng nhất thức:

suy ra:

.Quay lại bài toán ban đầu ta áp dụng bổ đề nhiều lần liên tiếp ta được(đây chỉ là kĩ năng tính toán nên mình chỉ ghi kết quả các bác thông cảm)

91

Page 92: Tong Hop Cac Dinh Li Hinh Hoc Phang Trong Cac Ki ThiHSG

suy ra:

(dpcm)

I.71)Định lí Poncelet về bán kính đường tròn nội tiếp,bàng tiếp trong tam giác vuông.Định lí:Cho tam giác có lần lượt là bán kính các đường tròn nội tiếp, bàng tiếp

góc .Chứng minh rằng: tam giác ABC vuông tại A khi và chỉ khi .

Chứng minh::Ta có:

tam giác vuông tại (dpcm)

I.72)Ðịnh lí HansenÐịnh lí:Cho tam giác .Chứng minh rằng các điều kiện sau tương đương:1)Tam giác vuông

2)

3)

Chứng minh:Ðầu tiên ta chứng minh một vài hệ thức phụ sau:

92

Page 93: Tong Hop Cac Dinh Li Hinh Hoc Phang Trong Cac Ki ThiHSG

1)

2)

3)Ta có:

Trở lại bài toán ban đầu ta có:

Do nên chia cả hai vế cho ta được"

(1)

Ðặt ta suy ra do dó (1)

tam giác vuông.

93

Page 94: Tong Hop Cac Dinh Li Hinh Hoc Phang Trong Cac Ki ThiHSG

(2)

tương tự thay như trên với chú ý và ta được:(2)

(3)

do lớn hơn nên (3) tam giác vuông.

*chú thích:

I.73)Định lí Steinbart mở rộngĐịnh lí:.Cho tam giác nội tiếp .Các tiếp tuyến của đường tròn tại giao nhau tại

.Trên (O) lấy các điểm .Chứng minh rằng đồng quy khi và chỉ khi đồng quy hoặc các giao điểm của với 3 cạnh tam giác thẳng hàng.

94

Page 95: Tong Hop Cac Dinh Li Hinh Hoc Phang Trong Cac Ki ThiHSG

Chứng minh:

Gọi

Ta có:

nên

95

Page 96: Tong Hop Cac Dinh Li Hinh Hoc Phang Trong Cac Ki ThiHSG

Tương tự ta suy ra:

do đó nếu vế phải bằng thì biểu thức trong ngoặc ở vế trái bằng hoặc và ngược lại hay nói cách khác đồng quy khi và chỉ khi đồng quy hoặc

thẳng hàng

I.74)Định lí Monge & d'Alembert IĐịnh lí:Cho 3 đường tròn có bán kính khác nhau và không chứa nhau.Tiếp tuyến chung ngoài của mỗi đường tròn giao nhau lần lượt tại .Chứng minh rằng: thẳng hàng.

Chứng minh:Vì các đường tròn có vai trò như nhau nên không mất tính tổng quát ta giả sử:

.Khi đó ta có thể chứng minh được:

Suy ra:

Theo định lí Menelaus ta suy ra dpcm

96

Page 97: Tong Hop Cac Dinh Li Hinh Hoc Phang Trong Cac Ki ThiHSG

*Chú thích:

là phép vị tự tâm tỉ số biến thành  

I.75)Định lí Monge & d'Alembert IIĐịnh lí:Cho 3 đường tròn có bán kính khác nhau và không chứa nhau.Tiếp tuyến chung trong của (A) và (C), (B) và (C) giao nhau lần lượt tại , tiếp tuyến

chung ngoài của và giao nhau tại .Chứng minh rằng: thẳng hàng.

Chứng minh:Vì các đường tròn có vai trò như nhau nên không mất tính tổng quát ta giả sử:

.Khi đó ta có thể chứng minh được:

Suy ra:

Theo định lí Menelaus ta suy ra dpcm

97

Page 98: Tong Hop Cac Dinh Li Hinh Hoc Phang Trong Cac Ki ThiHSG

*Chú thích:

là phép vị tự tâm tỉ số biến thành

I.76)Định lí Steiner về bán kính các đường trònĐịnh lí:Chứng minh rằng trong tam giác ta có:

Chứng minh:Ta có:

(hiển nhiên) suy ra dpcm

I.77)Định lí BellavitisĐịnh lí::Cho tứ giác là tứ giác điều hoà kí hiệu

.Chứng minh rằng:

98

Page 99: Tong Hop Cac Dinh Li Hinh Hoc Phang Trong Cac Ki ThiHSG

Chứng minh:Gọi đường tròn đường kính là đường tròn Apollonius của tam giác ứng với đỉnh .Vì là tứ giác điều hoà nên do đó thuộc đường tròn Apollonius của tam giác .Dựng đối xứng với qua phân giác .Ta suy ra:

Vậy (dpcm)

I.78)Định lí Feuer bach-Luchterhand:  

II/Một số điểm và đường đặc biệt được xác định duy nhất với tam giác và tứ giácII.1) Đường thẳng Euler của tam giác.

Định lí:Cho tam giác gọi là trực tâm, trọng tâm, tâm đường tròn ngoại tiếp.Chứng minh rằng: thẳng hàng.

99

Page 100: Tong Hop Cac Dinh Li Hinh Hoc Phang Trong Cac Ki ThiHSG

Chứng minh:Gọi là trung điểm .Ta có

do đó suy ra thẳng hàng

II)Đường tròn và tâm Euler

Kết quả : Trong một tam giác ,trung điểm các cạnh của tam giác ,chân các đường cao và trung điểm các đoạn thẳng nối trực tâm với các đỉnh cùng nằm trên một đường tròn gọi là đường tròn Euler của tam giác ấy.

Chỉ dẫn chứng minh:

Thực ra đây chỉ là một trường hợp đặc biệt của hai điểm đẳng giác (Xem mục I.50)

100

Page 101: Tong Hop Cac Dinh Li Hinh Hoc Phang Trong Cac Ki ThiHSG

II.3)Đường đối trung, điểm LemoineKết quảCho tam giác thì 3 đường đối trung của tam giác đồng quy tại điểm của tam giác.

Chỉ dẫn chứng minh::Đường đối trung của tam giác ứng với 1 đỉnh là đường thẳng đối xứng với trung tuyến qua phân giác tương ứng của đỉnh đó.Từ định nghĩa trên áp dụng định lí Xeva dạng sin ta có:

Suy ra đồng quy tại điểm của tam giác.

*Chú thích: kí hiệu tương đương với

II.4)Điểm Gergonne,điểm Nobb, đường thẳng Gergone

1)Kết quả về điểm Gergonne:Tam giác ABC với đường tròn nội tiếp (I).Tiếp điểm của (I) trên BC,CA,AB lần lượt là D,E,F.Khi đó AD,BE,CF đồng quy tại một điểm gọi là điểm Gergonne của tam giác ABC.

101

Page 102: Tong Hop Cac Dinh Li Hinh Hoc Phang Trong Cac Ki ThiHSG

Chỉ dẫn chứng minh:

Chỉ cần dùng định lí Ceva và các kết quả đơn giản : DB=DC,EA=EC,FA=FB là ra.

2)Kết quả về điểm Nobb và đường thẳng Gergonne(Vẫn với các kí hiệu trên)Một tam giác không cân có 3 điểm Nobb tương ứng là giao điểm của các cặp đường thẳng EF và CB ,DE và AB ,DF và AC. Và 3 điểm Nobb cùng nằm trên một đường thẳng gọi là đường thẳng Gergonne của tam giác ABC.

Chỉ dẫn chứng minh:

Xét cực và đối cực đối với (I). Đường đối cực của A là EF đi qua M,nên đường đối cực của M đi qua A.Mặt khác dễ thấy đường đối cực của M đi qua D nên suy ra đường đối cực của M là AD. Hoàn toàn tương tự ta có: Đường đối cực của N là BE và đường đối cực của P là CF

Theo trên ,do AD,BE,CF đồng quy nên sẽ có điều phải chứng minh.Bình luận: Kết quả trên có thể mở rộng như sau:

Cho tam giác ABC và 3 điểm D,E,F theo thứ tự thuộc BC,CA,AB sao cho AD,BE,CF đồng quy và D,E,F khác trung điểm đoạn thẳng.Gọi M,N,P lần lượt là điểm chung của các cặp đường thẳng (EF,BC) ,(DF,CA) ,(DE,AB).Khi đó M,N,P thẳng hàng

Bạn có thể chứng minh kết quả trên bằng định lí Menelaus nhưng thậm chí bài toán mở rộng này cũng chỉ là trường hợp đặc biệt của định lí Desargues mà thôi!!!!

(Xem them hai file : FG200821.bdf ; jcgeg200722.bdf)

II.5)Điểm NagelKết quả:.Cho tam giác . Các đường tròn bàng tiếp xúc với 3 cạnh tương ứng đỉnh lần lượt

tại thì ta có 3 đường thẳng đồng quy tại điểm của tam giác.

102

Page 103: Tong Hop Cac Dinh Li Hinh Hoc Phang Trong Cac Ki ThiHSG

Chỉ dẫn chứng minh:Ta có:

Suy ra dpcm *Chú thích:

II.6) Điểm Brocard

Định nghĩa:Trong một tam giác ABC cho trước có hai điểm Brocard M,N được xác định sao cho:

và .

II.7)Điểm SchifflerĐịnh nghĩa:Cho tam giác có là tâm đường tròn nội tiếp tam giác.Khi đó 4 đường thẳng

Euler của tam giác và đồng quy tại điểm của tam giác.

103

Page 104: Tong Hop Cac Dinh Li Hinh Hoc Phang Trong Cac Ki ThiHSG

Chỉ dẫn chứng minh:Gọi là tâm đường tròn ngoại tiếp tam giác, là trọng tâm tam giác, là trung điểm

là trọng tâm tam giác cắt tại cắt tại tiếp xúc với cạnh tại cắt tại , cắt tại .Rõ ràng là tâm đường tròn ngoại tiếp tam giác .Do đó là đường thằng Euler của tam giác .Áp dụng định lí Menelaus cho tam giác với cát tuyến ta có:

.Suy ra:

Ap dụng định lí Menelaus cho tam giác với cát tuyến ta có:

Do là trọng tâm tam giác IBC nên

Do đó:

Tương tự ta thấy các đường thẳng Euler của các tam giác cũng cắt tại (được

xác định bởi hệ thức )Vậy các đường thẳng Euler của 4 tam giác và đồng quy tại

(Xem them FG200312.bdf)

104

Page 105: Tong Hop Cac Dinh Li Hinh Hoc Phang Trong Cac Ki ThiHSG

II.8)Điểm Feuerbach

Kết quả:Trong một tam giác ,đường tròn Euler tiếp xúc với đường tròn nội tiếp của nó,và tiếp điểm đó được gọi là điểm Feuerbach của tam giác trên.

Chỉ dẫn chứng minh:(leductam post)Gọi là tâm đường tròn Euler, ngoại tiếp, nội tiếp của tam giác . là trực tâm,

là đường kính vuông góc với là hình chiếu của lên là bán kính đường tròn ngoại tiếp, nội tiếp. là chân đường phân giác góc

1.Đường tròn Euler tiếp xúc trong với đường tròn nội tiếp.Ta dễ dàng chứng minh được Vì suy ra (1)

105

Page 106: Tong Hop Cac Dinh Li Hinh Hoc Phang Trong Cac Ki ThiHSG

Chứng minh tiếp: Vì nên mà cân tại do đó Vậy Chiếu hệ thức trên lên theo phương vuông góc với ta được:

(2)Từ (1) và (2) ta suy ra:

(3)Ta có:

(4)

Từ (3) và (4) ta có:

Vậy suy ra dpcm.

Hoàn toàn tương tự ta cũng có:

2.Đường tròn Euler tiếp xúc ngoài với các đường tròn bàng tiếpTừ kẻ vuông góc với do nên

Từ (2) ta có:

Vậy (5)Mặt khác: nên:

(6)Từ (5) và (6) ta có: (7)

Từ kẻ . Trong tam giác vuông ta có:

(8)Từ (7) và (8) ta có:

106

Page 107: Tong Hop Cac Dinh Li Hinh Hoc Phang Trong Cac Ki ThiHSG

Vậy suy ra dpcm

(Xem them BalticFeuer; GenFeuerPDF; FG200117)

II.4)Điểm Gergonne,điểm Nobb, đường thẳng Gergone

1)Kết quả về điểm Gergonne:Tam giác ABC với đường tròn nội tiếp (I).Tiếp điểm của (I) trên BC,CA,AB lần lượt là D,E,F.Khi đó AD,BE,CF đồng quy tại một điểm gọi là điểm Gergonne của tam giác ABC.

Chỉ dẫn chứng minh:

Chỉ cần dùng định lí Ceva và các kết quả đơn giản : DB=DC,EA=EC,FA=FB là ra.

2)Kết quả về điểm Nobb và đường thẳng Gergonne(Vẫn với các kí hiệu trên)Một tam giác không cân có 3 điểm Nobb tương ứng là giao điểm của các cặp đường thẳng EF và CB ,DE và AB ,DF và AC. Và 3 điểm Nobb cùng nằm trên một đường thẳng gọi là đường thẳng Gergonne của tam giác ABC.

Chỉ dẫn chứng minh:

Xét cực và đối cực đối với (I). Đường đối cực của A là EF đi qua M,nên đường đối cực của M đi qua A.Mặt khác dễ thấy đường đối cực của M đi qua D nên suy ra đường đối cực của M là AD. Hoàn toàn tương tự ta có: Đường đối cực của N là BE và đường đối cực của P là CF

Theo trên ,do AD,BE,CF đồng quy nên sẽ có điều phải chứng minh.Bình luận: Kết quả trên có thể mở rộng như sau:

Cho tam giác ABC và 3 điểm D,E,F theo thứ tự thuộc BC,CA,AB sao cho AD,BE,CF đồng quy và D,E,F khác trung điểm đoạn thẳng.Gọi M,N,P lần lượt là điểm chung của các cặp đường thẳng (EF,BC) ,(DF,CA) ,(DE,AB).Khi đó M,N,P thẳng hàng

107

Page 108: Tong Hop Cac Dinh Li Hinh Hoc Phang Trong Cac Ki ThiHSG

Bạn có thể chứng minh kết quả trên bằng định lí Menelaus nhưng thậm chí bài toán mở rộng này cũng chỉ là trường hợp đặc biệt của định lí Desargues mà thôi!!!!

II.9)Điểm KosnitaĐịnh nghĩa.Cho tam giác là tâm đường tròn ngoại tiếp tam giác.Gọi là tâm các đường tròn ngoại tiếp tam giác .Khi đó ba đường thẳng và

đồng quy tại điểm của tam giác.

Chỉ dẫn chứng minh:Gọi tương ứng là giao điểm của với Ta có:

Tương tự với điểm và rồi sau đó nhân các tỉ lệ thức với nhau ta được dpcm

II.8)Điểm Feuerbach

Kết quả:Trong một tam giác ,đường tròn Euler tiếp xúc với đường tròn nội tiếp của nó,và tiếp điểm đó được gọi là điểm Feuerbach của tam giác trên.

108

Page 109: Tong Hop Cac Dinh Li Hinh Hoc Phang Trong Cac Ki ThiHSG

Chỉ dẫn chứng minh:(leductam post)Gọi là tâm đường tròn Euler, ngoại tiếp, nội tiếp của tam giác . là trực tâm,

là đường kính vuông góc với là hình chiếu của lên là bán kính đường tròn ngoại tiếp, nội tiếp. là chân đường phân giác góc

1.Đường tròn Euler tiếp xúc trong với đường tròn nội tiếp.Ta dễ dàng chứng minh được Vì suy ra (1)Chứng minh tiếp: Vì nên mà cân tại do đó Vậy Chiếu hệ thức trên lên theo phương vuông góc với ta được:

(2)Từ (1) và (2) ta suy ra:

(3)Ta có:

109

Page 110: Tong Hop Cac Dinh Li Hinh Hoc Phang Trong Cac Ki ThiHSG

(4)

Từ (3) và (4) ta có:

Vậy suy ra dpcm.Hoàn toàn tương tự ta cũng có:2.Đường tròn Euler tiếp xúc ngoài với các đường tròn bàng tiếpTừ {I}_{a} kẻ {I}_{a}{X}_{a} vuông góc với BC do {I}_{a}S = SI nên {X}_{a}M = MNTừ (2) ta có:

Vậy (5)Mặt khác: nên:

(6)Từ (5) và (6) ta có: (7)

Từ kẻ . Trong tam giác vuông ta có:

(8)Từ (7) và (8) ta có:

Vậy suy ra dpcm

Bổ sung một chứng minh khác bằng phép nghịch đảo ạ:

Xét có:

+ Đường tròn nội tiếp tiếp xúc với theo thứ tự tại .

+ Đường tròn bàng tiếp trong góc tiếp xúc với tại .

+ Đường tròn Euler qua trung điểm 3 cạnh là .

Kẻ tiếp tuyến chung của và tiếp xúc với chúng lần lượt tại . (chú ý là

và ). Gọi là tâm vị tự trong của và .

Ta có:

110

Page 111: Tong Hop Cac Dinh Li Hinh Hoc Phang Trong Cac Ki ThiHSG

Nếu thì hiển nhiên đã tiếp xúc với và nên ta chỉ quan tâm đến trường hợp .

Khi đó, nên theo hệ thức Newton, .

Đặt , lại có nên suy ra

.

Cuối cùng, xét phép nghịch đảo cực , phương tích

. Nhưng tiếp xúc với

và còn bản thân 2 đường tròn này bất biến qua phép nghịch đảo đang xét nên ta cũng có

cũng tiếp xúc với và .

Lập luận tương tự cho thấy tiếp xúc với .

Kết thúc chứng minh!

Bổ Đề:1, tam giác là phân giác. Điểm nằm trong mặt phẳng tam giác thì

2, tứ giác nội tiếp được, khi đó phân giác góc và đồng quy.  

II.10)Điểm Musselman,định lí Paul Yiu về điểm MusselmanKết quả: Cho tam giác . Các điểm lần lượt là điểm đối xứng với qua các

cạnh đối diện và là tâm đường tròn ngoại tiếp. Khi đó và cùng đi qua 1 điểm là ảnh của điểm qua phép nghịch đảo đường tròn ngoại tiếp tam giác

.Chỉ dẫn chứng minh:

111

Page 112: Tong Hop Cac Dinh Li Hinh Hoc Phang Trong Cac Ki ThiHSG

Gọi và lần lượt là ảnh của điểm và qua phép nghịch đảo đường tròn ngoại tiếp tam giác .Khi đó ta có:

Suy ra

Bây giờ ta cần chứng minh 3 điểm , và thẳng hàng.

Ta có bộ tâm tỉ cự của điểm

và tâm đường tròn chín điểm do đó điểm và tâm đường tròn chín điểm là 2 điểm đẳng giác của tam giác

Mặt khác - tâm đường tròn ngoại tiếp và - trực tâm cũng là 2 điểm đẳng giác của tam giác

nên suy ra

Ta cũng dễ dàng chứng minh được

Vậy hay nói cách khác 4 điểm và cùng nằm trên 1 đường tròn

Tương tự với các đường tròn còn lại ta suy ra dpcm Điểm được gọi là điểm của tam giác

Định lí Paul Yiu về điểm Musselman: Với giả thiết như trên thì 3 đường tròn

và cũng đi qua điểm .Chỉ dẫn chứng minh:

112

Page 113: Tong Hop Cac Dinh Li Hinh Hoc Phang Trong Cac Ki ThiHSG

Về định lí này em xin trích dẫn trực tiếp lời giải của Darij Grinberg:

Theo chứng minh trên thì điểm nằm trên và nên ta được:

Ta có:

Mặt khác:

Vậy hay và cùng nằm trên đường tròn.Tương tự với các đường tròn khác ta suy ra dpcm

113

Page 114: Tong Hop Cac Dinh Li Hinh Hoc Phang Trong Cac Ki ThiHSG

II.11)Khái niệm vòng cực của tam giác.Khái niệm::Cho tam giác tù . Chân các đường cao đối diên các đỉnh là và .Khi đó vòng cực của tam giác là đường tròn có tâm là và bán kính xác định với Trong đó: - bán kính đường tròn ngoại tiếp

- độ dài 3 cạnh tam giác- 3 góc tam giác

Ngoài ra ta có 1 tính chất của vòng cực là: cho 3 điểm bất kì chuyển động trên các đường cạnh của tam giác ABC dựng các đường tròn có đường kính là đoạn thẳng nối 1 đỉnh với điểm chuyển động trên cạnh đối diện thì khi đó vòng cực của tam giác trực giao với tất cả các đường tròn đó.

II.12)Điểm GibertKết quả:: Cho tam giác và điểm - ảnh của điểm qua phép nghịch đảo đường tròn ngoại tiếp tam giác . Gọi là điềm đối xứng với điểm

qua các đường cạnh của tam giác. Khi đó các đường thẳng và đồng quy tại điểm nằm trên đường tròn ngoại tiếp tam giác

Chỉ dẫn chứng minh:

114

Page 115: Tong Hop Cac Dinh Li Hinh Hoc Phang Trong Cac Ki ThiHSG

Gọi giao điểm của và là thì ta có:

Vậy nằm trên đường tròn ngoại tiếp tam giác ABC.Chứng minh tương tự với và ta suy ra dpcm

II13/Trục LemoineĐịnh lý:

Cho tam giác ABC nội tiếp đường tròn .Tiếp tuyến tại A của đường tròn cắt đường thẳng BC tại X.Định nghĩa tương tự cho Y,Z.Chứng minh rằng X,Y,Z thẳng hàng và đường thẳng chứa X,Y,Z được gọi là trục Lemoine của tam giác ABC

115

Page 116: Tong Hop Cac Dinh Li Hinh Hoc Phang Trong Cac Ki ThiHSG

Chứng minh:

Không mất tính tổng quát giả sử hai vecto và cùng hướng.Do hai tam giác ABX và CXA đồng dạng với nhau suy ra:

Nhân hai đẳng thức trên với nhau suy ra:

Do X nằm ngoài tam giác nên suy ra:

Tương tự ta có hai đẳng thức sau:

Nhân ba đẳng thức với nhau ta có:

116

Page 117: Tong Hop Cac Dinh Li Hinh Hoc Phang Trong Cac Ki ThiHSG

Theo định lý Menelaus thì ba điểm X,Y,Z đồng quy.

Chứng minh (2):

Ta sẽ chứng minh đường đối cực của X,Y,Z đối với lần lượt là ba đường đối trung của tam giác ABC.Edit later...  

II.14)Tâm MorleyKết quả::Tâm thứ nhất được định nghĩa là tâm đường tròn ngoại tiếp của tam giác

Morley thứ nhất. Tâm thứ hai được định nghĩa là tâm phối cảnh của tam giác với tam giác Morley thứ nhất.Chỉ dẫn chứng minh:

Áp dụng định lí tri Xeva cho 3 đường thẳng đồng quy là và ta có:

Suy ra

Tương tự với các đường còn lại ta được dpcm

II.15) Tâm Spieker và đường thẳng NagelKết quả:Cho tam giác . Gọi lần lượt là trung điểm của và . Tâm

đường tròn nội tiếp tam giác là tâm của tam giác ABC.Khi đó 4 điểm tâm

đường tròn nội tiếp, trọng tâm, điểm và tâm cùng nằm trên đường thẳng

của tam giác

117

Page 118: Tong Hop Cac Dinh Li Hinh Hoc Phang Trong Cac Ki ThiHSG

Chỉ dẫn chứng minh:

Ta có phép vị tự tâm tỉ số biến tam giác thành tam giác nên suy ra

(1)

Xét điểm có bộ số tâm tỉ cự:

(2)Từ (1) và (2) ta suy ra điểm và cùng nằm trên 1 đường thẳng.Ngoài ra ta còn 1 vài tính chất của điểm Spieker như nó là tâm đẳng phương của 3 đường tròn bàng tiếp tam giác ABC, tâm Spieker, điểm Brocard thứ 3 và điểm đẳng cự với tâm đường tròn nội tiếp thẳng hàng...

II.16)Hai điểm FermatKết quả:Cho tam giác . Dựng ra phía ngoài(vào trong) các tam giác đều và

. Khi đó tâm phối cảnh của tam giác và tam giác được gọi là điểm thứ nhất(thứ hai) hay người ta còn gọi là điểm dương(âm).

Chỉ dẫn chứng minh:

118

Page 119: Tong Hop Cac Dinh Li Hinh Hoc Phang Trong Cac Ki ThiHSG

Gọi giao điểm của và lần lượt với và là ta có:

Tương tự với và ta được:

Mặt khác:

Tương tự ta suy ra dpcm Ngoài ra:

119

Page 120: Tong Hop Cac Dinh Li Hinh Hoc Phang Trong Cac Ki ThiHSG

Trong đó: là lần diện tích tam giác

Xem them

The Fermat Point and Generalizations

P. Fermat (1601-1665) challenged Evangelista Torricelli(*) (1608-1647), the inventor of

barometer with the following question

 Find the point such that the sum of its distances from the vertices of a triangle is a

minimum.

Torricelli presented several solutions. In one he observed that the circumcircles of the

equilateral triangles constructed externally on the sides of a given triangle meet in a point.

Many more have been found since. I'll present several solutions and two quite surprising

generalizations.

Solution 1

(Published by Joseph Ehrenfried Hofmann (1900--1973) in 1929. It was independently

discovered by Tibor Galai and others [Honsberger, p. 26].)

In ΔABC, select a point P and connect it with vertices A, B, and C. Rotate ΔABP 60o around B

into position C'BP'. By construction, ΔBPP' is equilateral, PB = P'B, and PA = C'P'. We thus have

PA + PB + PC = C'P' + P'P + PC. As the image of A under the rotation, position of C' does not

depend on P. Also, PA + PB + PC CC' because the broken line CPP'C' is no shorter than the

straight line CC'. Therefore, PA + PB + PC reaches its minimum iff P lies on CC'. For this P,

BPC' = 60o. Had we rotated ΔABP around A, we would have found that APC' = 60o.

The result is clearly related to Napoleon's theorem. On the sides of ΔABC construct equilateral

triangles ABC', ACB', and BCA'. We know that the Fermat point P that minimizes the sum PA +

120

Page 121: Tong Hop Cac Dinh Li Hinh Hoc Phang Trong Cac Ki ThiHSG

PB + PC lies on CC'. By the same token, it lies on AA' and BB'. Therefore, it lies on their

intersection. As far as Napoleon's theorem goes, the three lines AA', BB', and CC' are

concurrent. (This was already known to Thomas Simpson (1710--1761).) Not only that but they

cross at angles equal 120o. Furthermore, AA' = BB' = CC' since each of them equals PA + PB +

PC for the Fermat point P.

So the Fermat point is unique and lies at the intersection of three straight lines that connect

vertices of the triangle with opposite vertices of Napoleon's triangles. The construction fails if

one of the internal angles of ΔABC is 120o or more. In this case, the vertex corresponding to the

largest angle of the triangle solves Fermat's problem.

(There is a dynamic illustration pertinent to the above proof.)

Solution 2

 

This solution only works under the assumption that the Fermat point exists and is unique. Let

for a given ΔABC, P be that point. Let's move, say, A a little into position A1 on the ray PA. Will P

move along into a different position? Assuming it will and denoting its new position as P1, we

arrive at contradiction. Indeed, for ΔABC,

 PA + PB + PC < P1A + P1B

+ P1C

Also, by our assumption for ΔA1BC,

121

Page 122: Tong Hop Cac Dinh Li Hinh Hoc Phang Trong Cac Ki ThiHSG

 P1A1 + P1B + P1C < PA1 + PB

+ PC

Summing up and cancelling, we get

 PA + P1A1 < PA1 +

P1A

Recollect that we selected A1 on the ray PA so that PA1 + A1A = PA. Which leads to

 AA1 + P1A1 <

P1A

which is of course absurd. Therefore, when A slides along the ray PA, point P does not change.

Slide then vertices A and B so as to make ΔABC equilateral. This is always possible. For

example, start with sliding A so as to make AB = BC. The triangle ABC becomes isosceles. It is

obvious that, for an isosceles triangle, the Fermat point lies on the axis of symmetry of the

triangle. For this reason, the triangle remains isosceles wherever B is located on that axis. Slide

it so as to make all three sides of ΔABC equal. For an equilateral triangle, its centroid that also

serves as the incenter and the circumcenter, serves as the Fermat point as well.

Solution 3

([Honsberger, p. 26] ascribes this solution to Torricelli himself and mentions that it was

rediscovered almost 300 years later by F. Riesz. In a private communication, Douglas Rogers

mentioned that the solution has been also proposed by J. Steiner. Thus the same object that is

most frequently referred to as Fermat's point, Torricelli's point and Fermat-Torricelli point, is

sometimes also named after J. Steiner, as Steiner's point, especially in the framework of

Steiner's networks. [Tikhomirov, p. 31] and [Courant and Robbins, p. 354] even refer to

Fermat's problem as Steiner's. [Johnson, p. 221] attributes the present solution to Steiner.)

122

Page 123: Tong Hop Cac Dinh Li Hinh Hoc Phang Trong Cac Ki ThiHSG

The following solution depends on explicit knowledge of the fact that, for Fermat's point, AFB

= BFC = AFC = 120o. Without proving that such a point exists, it shows that, if it does, it

solves Fermat's problem.

 

At the vertices of ΔABC draw perpendiculars to lines AF, BF, and CF. The new lines form an

equilateral triangle RST. Let F1 be a point different from F. Drop perpendiculars F1A1, F1B1 and

F1C1 to the sides of ΔRST. It's a nice property (known as Viviani's theorem) of equilateral

triangles that in ΔRST,

 FA + FB + FC = F1A1 + F1B1 +

F1C1.

(The sum of the distances from a point inside an equilateral triangle to the sides of the triangle

does not depend on the point. )

On the other hand, obviously,

  F1A1 + F1B1 + F1C1 < F1A + F1B

123

Page 124: Tong Hop Cac Dinh Li Hinh Hoc Phang Trong Cac Ki ThiHSG

+ F1C

All that remains is to combine the two.

(Douglas Rogers made a delightful observation. ΔRST is the largest equilateral triangle

circumscribing ΔABC. According to Viviani's theorem, FA, FB, FC add up to the altitude of ΔRST.

For any other equlateral ΔLMN curcumscribing ΔABC, at least one of FA, FB, or FC is not

perpendicular to the side and hence is longer than such perpenduclar. It follows that the sum

FA + FB + FC is bound to be larger than the alititude of ΔLMN. This implies that the side of

ΔLMN is smaller than that of ΔRST. In passing, ΔRST is known as the antipedal triangle of F with

respect two ΔABC.)

Solution 4

(The proof by Lou Talman is found on one of the discussions at the mathforum. Still very

simple, it uses a little bit of analytic geometry and calculus. This is probably very close to one

of Torricelli's original proofs.)

One can see that the Fermat point does minimize FA + FB + FC as follows: Let z be the sum FA

+ FB, where F is chosen to minimize FA + FB + FC, and consider the locus of points P that

satisfy PA + PB = z. This locus is an ellipse with foci at A and B. Because F minimizes z + FC,

the line determined by C and F must be normal to the tangent to this ellipse at F. By the

reflection property of the ellipse, the angles AFC and BFC must be equal. A similar argument

shows that the angles AFC (say) and AFB must also be equal. From this it follows that the three

angles are all 120 degrees.

Solution 5

(The argument is similar to that in Solution 4, but from a different perspective. [F. G.-M., p. 442]

credits Lhuilier (1811) with the proof.)

124

Page 125: Tong Hop Cac Dinh Li Hinh Hoc Phang Trong Cac Ki ThiHSG

Assume FC is constant. Then point F that minimizes FA + FB + FC, minimizes also FA + FB and

lies on the circle with center C and radius FC. The minimum is achieved for a point F on the

circle for which the angles AFC and BFC are equal.

 

For more detail, draw a tangent to the circle at F and choose any point G on the tangent other

than F. Let H be the intersection of HC with the circle. Then, assuming angles AFC and BFC

equal,

 FA + FB < GA + GB < HA

+ HB

(The latter inequality warrants extra attention. It follows from the fact the various ellipses with

foci at A and B do not intersect.) From here,

 FA + FB + FC < HA + HB + FC = HA +

HB + HC.

The argument can be repeated assuming either FA or FB constant. As a result, at Fermat's

point F all three angles AFC, BFC and AFB are found to be equal.

125

Page 126: Tong Hop Cac Dinh Li Hinh Hoc Phang Trong Cac Ki ThiHSG

Solution 6

Fermat's point can be located with the help of Euler's generalization of Ptolemy's Theorem, see

[Pedoe, pp. 93-94].

 

Any triangle has at least two acute angles. Given ΔABC, let angles at B and C be acute. Form an

equlateral triangle BCD, with D and A on opposite sides of BC. Let (O) be the circumcircle of

ΔBCD.

For a point Q, by Ptolemy's inequality

 BQ·CD + CQ·BD ≥

DQ·BC,

with equality only if Q lies on (O) and such that the quadrilateral BQCD is convex. Note that, by

construction, BC = CD = BD which reduces the above to

  BQ + CQ ≥

126

Page 127: Tong Hop Cac Dinh Li Hinh Hoc Phang Trong Cac Ki ThiHSG

DQ.

Therefore

 AQ + BQ + CQ ≥ AQ + DQ

≥ AD.

Unless, Q lies on AD, AQ + BQ + CQ > AD. Let P be the intersection of AD with (O) other than D.

BPCD is an inscribed convex quadrilateral and P lies on AD. So in this (and only in this) case AP

+ BP + CP = AD. For any other selection of Q,

 AQ + BQ + CQ > AP + BP

+ CP.

Angle BPC is supplementary to BDC = 60° so that BPC = 120°. Further, D is in the middle

of the arc BDC so that angles BPD and CPD are 60° making angles APC and APB both equal

120°.

If the triangle is acute the same construction applies to the other two sides which brings up the

framework of Napoleon's theorem. The three circles intersect at Fermat's point and three lines

joining the vertices of ΔABC with the opposite vertices of the Napoleon triangles concur at the

point P.

The latter fact can be used for a more direct proof.

Solution 7

(This was a part of a solution by Grimbal to a problem posted at the wu::forum.)

127

Page 128: Tong Hop Cac Dinh Li Hinh Hoc Phang Trong Cac Ki ThiHSG

 

Construct Napoleon's equilateral triangles ABC', AB'C', A'BC externally on the sides of ΔABC. Let

P be the intersection of AA' and BB'. AA' is CC' rotated clockwise 60° about B,

and BB' is CC' rotated counterclockwise 60° about A, it follows that angle APB'

is 60°. Let X be the point on BB' making triangle APX equilateral. Now, if BB' is

rotated clockwise 60° about A, then X goes to P, B' to C, and B to C'. Hence P,

C, and C' are collinear; and so AA', BB', and CC' are concurrent, and any two of

these lines make an angle of 120°.

Searching for the Fermat point we discovered a nice property of Napoleon's triangles. I found

quite surprising generalizations of those properties in a column by David Gale in The

Mathematical Intelligencer which was kindly pointed out to me by Professor McWorter.

Two lines passing through a vertex of a triangle are called isogonal with respect to that vertex

if they form equal angles with its (internal) angle bisector. Following is the first generalization.

Theorem 1

128

Page 129: Tong Hop Cac Dinh Li Hinh Hoc Phang Trong Cac Ki ThiHSG

As in the diagram, assume lines AB' and AC' are isogonal as are pairs CB', CA' and BA', BC'.

Then three lines AA', BB', and CC' are concurrent, i.e., meet at a common point.

This applet requires Sun's Java VM 2 which your browser may perceive

as a popup. Which it is not. If you want to see the applet work, visit

Sun's website at http://www.java.com/en/download/index.jsp, download

and install Java VM and enjoy the applet.

Buy this applet

What if applet does not run?

Napoleon's theorem is obtained when all three angles involved are equal to 30o. The common

point is then known as the First Napoleon point. (The Second Napoleon Point is obtained when

the equlateral triangles are formed internally to the given triangle. Similarly, there are two

Fermat's points, which are also known as the first and second isogonic centers.) If the base

angles are just equal between themselves, the theorem bears

the name of Ludwig Kiepert who replaced equilateral

triangles with isosceles ones. An interesting specification of

the theorem is obtained when the three angles add up to

180o. But the theorem also admits further generalization.

In Theorem 1, pairs of (isogonal) lines were related to angle

bisectors. As is well known from elementary geometry, angle

bisectors meet at a single point (incidentally, the incenter of the triangle.)

Two lines AB' and AC' through a vertex A of a triangle are said to be isotomic if they intersect

the opposite side BC in points equidistant from its midpoint Ma. Theorem 1 remains valid for

isotomic lines as well. And, in a certain sense, for any three concurrent lines.

129

Page 130: Tong Hop Cac Dinh Li Hinh Hoc Phang Trong Cac Ki ThiHSG

A second generalization that I am about to formulate and then prove

belongs to the realm of Projective Geometry.

Theorem 2

Let p, q, r be concurrent lines through the vertices A, B, and C, respectively, of ΔABC. Let PA be

the pencil of lines at A and let TA be the (unique) projective mapping on PA which

1. interchanges lines AB and AC,

2. leaves p fixed.

Define PA, PB, and TA, TB similarly. For any line a in PA, let a' = TA(a). Similarly, for b in PB and c in

PC, let b' = TB(b) and c' = TC(c). Let C' = a' b, B' = c' a, and A' = b' c.

Then AA', BB', and CC' are concurrent.

Note that (an apparently more general) Theorem 2 follows from Theorem 1 with a projective

mapping that leaves vertices A,B,C fixed but takes the incenter into any point P (the

intersection of lines p, q, r.)

To prove Theorem 2, perform the projective mapping that carries C to the origin, A to the point

at infinity at y-axis, B to the point at infinity at x-axis, and P to the point (1, 1).

Lines p, q, and r are carried onto the lines x = 1, y = 1, and x = y, respectively. We thus have

 

PA is the set of all vertical lines,

PB is the set of all horizontal lines,

PC is the set of all lines through the

origin.

Transformation TA preserves the vertical direction and, therefore, is in reality a 1-dimensional

projective transformation. So it's a Möbius transformation which, in general, has the form

f(x) = (ax + b)/(cx + d). Transformations that interchange x = 0 with the point at infinity are

130

Page 131: Tong Hop Cac Dinh Li Hinh Hoc Phang Trong Cac Ki ThiHSG

given by f(x) = a + b/x (c = 1 and d = 0.) Among those, there is a single one that leaves x = 1

fixed: f(x) = 1/x. So TA maps x = a onto x = 1/a.

To simplify the notations, we will denote the vertical line with x = a by a, the horizontal line

y = b by b, and the line y = cx through the origin, by c. Then with similar definitions for TB and

TC we have

 a' = TA(a) = 1/a, b' = TB(b) = 1/b, and c'

= TC(c)

Further

 

C' = a' b = (1/a, b),

B' = c' a = (a, a/c),

and

A' = b' c = (1/(bc),

1/b).

Thus AA' has equation x = 1/(bc), BB' has equation y = a/c, and CC' has equation y = (ab)x.

Then, AA' BB' = (1/(bc), a/c) which lies on the line CC'.

II.17)Điểm Parry reflection.Kết quả:: Cho tam giác . Kẻ qua các đường thẳng song song với nhau và

song song với đường thẳng Euler của tam giác. Gọi lần lượt là các đường đối xứng với

qua và . Khi đó các đường này đồng quy tại điểm Parry reflection của tam giác ABC.Chỉ dẫn chứng minh:

131

Page 132: Tong Hop Cac Dinh Li Hinh Hoc Phang Trong Cac Ki ThiHSG

Gọi là trực tâm tam giác , là ảnh của qua phép đối xứng trục và . Phép vị tự tâm tỉ số biến tam giác thành tam giác khi đó 2

đường thẳng của 2 tam giác này trùng nhau. Gọi là giao điểm của với và Ta có:

Vậy cũng là ảnh của qua phép đối xứng trục Dựa vào tính chất đồng dạng ta suy ra được đường thẳng đối xứng với đường thẳng qua trục

Tương tự với các đường thẳng còn lại ta suy ra 3 đường thẳng đồng quy theo định lí

(Xem them File FG200806.bdf)

132

Page 133: Tong Hop Cac Dinh Li Hinh Hoc Phang Trong Cac Ki ThiHSG

II.18)Đường tròn Taylor ,tâm TaylorKết quả:.Cho tam giác các đường cao . Từ kẻ các đường vuông góc với các cạnh khi đó chân các đường vuông góc này nằm trên cùng 1 đường tròn gọi là

đường tròn của tam giác Chỉ dẫn chứng minh:

Ta có: Vậy 4 điểm cùng nằm trên 1 đường tròn.Tương tự với 2 bộ 4 điểm và .

Ta được: Vậy 4 điểm cũng cùng nằm trên 1 đường tròn hay ta suy ra được dpcm

II.19)Điểm Bevan Kết quả:. Cho tam giác , là tâm các đường tròn bàng tiếp. Khi đó tâm đường tròn ngoại tiếp tam giác được gọi là điểm của tam giác .

133

Page 134: Tong Hop Cac Dinh Li Hinh Hoc Phang Trong Cac Ki ThiHSG

Sau đây ta sẽ đến với 1 vài tính chất cua điểm : *Ta thấy rằng là tâm đường tròn nội tiếp tam giác chính là trực tâm của tam giác

khi đó đường tròn ngoại tiếp tam giác ABC là đường tròn chín điểm của tam giác suy ra là trung điểm của

*Tâm là trung điểm của đoạn nối trực tâm với điểm .Đầu tiên ta chứng minh 3 điểm này thẳng hàng.Xét tam giác ta có:

Vậy 3 điểm này thẳng hàng.Xét tam giác ta có 3 điểm thẳng hàng suy ra:

Vậy ta có dpcm

*Điểm là trung điểm của đoạn nối điểm và điểm .

Điểm là điểm đối xứng của trực tâm qua tâm đường tròn ngoại tiếp.Khi đó tương tự như trên ta cũng có được dpcm

134

Page 135: Tong Hop Cac Dinh Li Hinh Hoc Phang Trong Cac Ki ThiHSG

 

II.20)Điểm VectenKết quả:. Cho tam giác . Dựng ra phía ngoài(hay vào trong) các hình vuông

và . Khi đó đường nối 1 đỉnh của tam giác với tâm hình vuông dựng trên cạnh đối diện đồng quy tại điểm Vecten của tam giác .Chỉ dẫn chứng minh:

Theo định lí thì điều này hiển nhiên và cũng dễ dàng suy ra được có 2 điểm trong và ngoài(hay âm và dương).

135

Page 136: Tong Hop Cac Dinh Li Hinh Hoc Phang Trong Cac Ki ThiHSG

Ngoài ra ta còn có 1 tính chất khá thú vị về điểm : Tâm đường tròn chín điểm và 2 điểm thẳng hàng.

II.21)Điểm MittenpunktKết quả: Cho tam giác , là tâm các đường tròn bàng tiếp, lần lượt là trung điểm các cạnh . Khi đó các đường thẳng đồng quy tại điểm

của tam giác .Chỉ dẫn chứng minh:

Ta có:

Chiếu hệ thức theo phương lên trục ta được:

Tương tự với các đường còn lại ta suy ra dpcm

Ta có:

Ngoài ra ta còn có 1 vài tính chất bên lề khá thú vị: Với giả thiết như trên gọi là điểm tiếp xúc của đường tròn nội tiếp tam giác với 3 cạnh thì đồng quy. Khi đó điểm

này và điểm là 2 điểm đẳng giác.

136

Page 137: Tong Hop Cac Dinh Li Hinh Hoc Phang Trong Cac Ki ThiHSG

*Điểm , tâm và trực tâm thẳng hàng

*Điểm , tâm đường tròn nội tiêp và điểm thẳng hàng.

*Điểm , trọng tâm va điểm thẳng hàng với

II.22)Điểm NapoleonKết quả:.Cho tam giác , dựng ra phía ngoài( hay vào trong) các tam giác đều

và . Khi đó đường nối 1 đỉnh của tam giác với trọng tâm tam giác đều dựng trên cạnh đối diện đồng quy tại điểm Napoleon của tam giác .Chỉ dẫn chứng minh:

Áp dụng định lí ta suy ra dpcm cũng dễ dàng suy ra có 2 điểm là trong và ngoài

II.23)Đường tròn Adam

Kết quả:Cho tam giác ABC với điểm Gergonne G.Đường thẳng qua G song song với EF cắt AB,AC ở S,P.Đường thẳng qua G song song với DE cắt AC,BC ở Q,M.Đường thẳng qua G song song với DF cắt BA,BC ở R,N.Khi đó các điểm M,N,P,Q,R,S cùng thuộc một đường tròn gọi là đường tròn Adam của tam giác ABC.

137

Page 138: Tong Hop Cac Dinh Li Hinh Hoc Phang Trong Cac Ki ThiHSG

Chỉ dẫn chứng minh:

Gọi (I) là đường tròn nội tiếp tam giác ABC,tiếp điểm của (I) trên BC,CA,AB lần lượt là D,E,F.Đường thẳng qua A và G song song với BC tương ứng cắt DE,DF ở (H,K),(V,T).Ta thấy:

.Do đó AH=AK nên GT=GV.Bây giờ để ý rằng GTDN và GVDM là hai hình bình hành nên ta cũng có DM=DN.Kết hợp với ID vuông góc với BC ta thu được IM=IN.Tương tự IP=IQ,IR=ISMặt khác dễ thấy :IM=IQ=IR.Từ các khẳng định trên ta dễ nhận được điều cần chứng minh.

II.24)Tam giác Fuhrmann ,đường tròn FuhrmannKết quả:Cho tam giác nội tiếp đường tròn tâm . Gọi là trung điểm các cung BC, CA, AB. Lấy các điểm trên đối xứng qua các cạnh tương ứng ta được 3 điểm nữa là

và .Khi ấy tam giác được gọi là tam giác của tam giác .

138

Page 139: Tong Hop Cac Dinh Li Hinh Hoc Phang Trong Cac Ki ThiHSG

Đường tròn ngoại tiếp tam giác được gọi là đường tròn

Tính chất:

1)

2)

3)Trực tâm của tam giác trùng với tâm đường tròn nội tiếp của tam giác .(để chứng minh ta có thế sử dụng tích vô hướng)

4)Tâm đường tròn chín điểm của tam giác và tam giác trùng nhau. Bán kính đường tròn ngoại tiếp tam giác có độ dài bằng

II.25)Hình luc giác và đường tròn Lemoine thứ nhấtKết quả:Cho tam giác và điểm . Qua kẻ các đường thẳng song song với các cạnh cắt các cạnh còn lại tại . Khi ấy lục giác được gọi là lục giác thứ nhất của tam giác .

139

Page 140: Tong Hop Cac Dinh Li Hinh Hoc Phang Trong Cac Ki ThiHSG

Tính chất:1)Lục giác thứ nhất là lục giác ngoại tiếp. Đường tròn ngoại tiếp lục giác này được gọi là đường tròn thứ nhất:Do là đường thẳng đối trung đi qua trung điểm của nên là đường đối song tương ứng cạnh . Suy ra tứ giác là tứ giác nội tiếp.

2)Các cạnh bị kẹp giữa các đường song song tỉ lệ với lũy thừa bậc ba cạnh tương ứng:Gọi x, y, z là khoảng cách từ L tới 3 cạnh tam giác. Khi đó:

3)3 đoạn trên cùng 1 cạnh tỉ lệ với bình phương các cạnh của tam giác

4)Tâm của đường tròn Lemoine thứ nhất là trung điểm đoạn nối điểm Lemoine với tâm đường tròn ngoại tiếp tam giác ABC:Để ý vuông góc với khi đó ta dễ dàng suy ra được dpcm.

5)Bán kính

Ngoài ra ta còn có cách viết khác:

Trong đó là bán kính đường tròn Lemoine thứ hai(sẽ được nói đến trong phần sau)

140

Page 141: Tong Hop Cac Dinh Li Hinh Hoc Phang Trong Cac Ki ThiHSG

II.26)Hình lục giác và đường tròn Lemoine thứ hai

Khái niệm về đường đối song:Cho tam giác . Với 2 điểm bất kì thuộc các cạnh và ta có 2 kiểu chọn sao cho tam giác đồng dạng với tam giác . Thứ

nhất là song song với Thứ hai là tứ giác nội tiếp như hình vẽ. Khi ấy và được gọi là các đường đối song tương ứng với góc

Tính chất:Đường đối trung luôn đi qua trung điểm của các đường đối song tương ứng với cùng 1 đỉnh.

Kết quả:Cho tam giác và điểm . Qua kẻ các đường đối song tương ứng với các cạnh của tam giác cắt các cạnh còn lại tại Khi đó lục giác

được gọi là lục giác thứ hai.

Tính chất:1)Lục giác thứ hai nội tiếp 1 đường tròn và đường tròn này được gọi là đường tròn

thứ hai của tam giác ABC.

141

Page 142: Tong Hop Cac Dinh Li Hinh Hoc Phang Trong Cac Ki ThiHSG

2)Bán kính

Thực chất lục giác và đường tròn chỉ là trường hợp đặc biệt của lục giác và đường tròn . Xem thêm trong

II.27)Điểm Euler của Tứ giác nội tiếpKết quả:Cho tứ giác nội tiếp . Gọi lần lượt là trực tâm các tam giác

và . Khi ấy thì các đường thẳng và đồng quy. Điểm đồng quy được gọi là điểm của tứ giác nội tiếp.

Chỉ dẫn chứng minh:Ta có song song và cùng bằng 2 lần khoảng cách từ tới nên tứ giác

là hình bình hành suy ra giao nhau tại trung điểm mỗi đường.Tương tự ta suy ra bốn đường thẳng đồng quy tại trung điểm mỗi đường.

Từ đây ta suy ra được nhiều tính chất thú vị của điểm

1)Điểm nằm trên đường vuông góc hạ từ trung điểm một cạnh tới cạnh đối diện(hoặc trung điểm đường chéo tới đường chéo còn lại).

2)Đường thẳng của đỉnh với tam giác thì đi qua điểm . Tương tự với các đỉnh còn lại.

3)Đường tròn chín điểm của các tam giác và đồng quy tại điểm

Xem thêm

II.28)Đường thẳng Steiner của tứ giác toàn phầnKết quả:Cho tứ giác toàn phần Khi đó trực tâm của các tam giác

142

Page 143: Tong Hop Cac Dinh Li Hinh Hoc Phang Trong Cac Ki ThiHSG

và cùng nằm trên 1 đường thẳng được gọi là đường thẳng của tứ giác toàn phần.

Chỉ dẫn chứng minh:Gọi lần lượt là trực tâm các tam giác và Gọi là trung điểm các đường chéo Khi đó:

Vậy nằm trên trục đẳng phương của 2 đường tròn và Tương tự ta cũng có 3 trực tâm còn lại cùng nằm trên trục đẳng phương của hai đường tròn này suy ra dpcm.

II.29)Đường thẳng Gauss của tứ giác toàn phần.Kết quả:Cho tư giác toàn phần . Khi đó trung điểm các đường chéo cùng nằm trên một đường thẳng được gọi là đường thẳng của tứ giác toàn phần.

143

Page 144: Tong Hop Cac Dinh Li Hinh Hoc Phang Trong Cac Ki ThiHSG

Chỉ dẫn chứng minh:Gọi lần lượt là trung điểm các đường chéo và

là tam giác trung bình của tam giác Khi đó các điểm nằm trên các cạnh của tam giác .Ta có:

Nhân các vế các đẳng thức trên ta được:

Suy ra dpcm.Từ 2 bài viết trên ta thấy rằng trong 1 tứ giác toàn phần thì đường thẳng Steiner vuông góc với đường thẳng Gauss.

II.30) Điểm Miquel của tứ giác toàn phầnKết quả:Cho tứ giác toàn phần . Khi ấy đường tròn ngoại tiếp của các tam giác

và đồng quy. Điểm đồng quy đó được gọi là điểm của tứ giác toàn phần.

144

Page 145: Tong Hop Cac Dinh Li Hinh Hoc Phang Trong Cac Ki ThiHSG

Chỉ dẫn chứng minh:Giả sử đường tròn ngoại tiếp các tam giác và giao nhau tại điểm khác Khi đó ta có:

Vậy nằm trên đường tròn ngoại tiếp tam giác . Tương tự với đường tròn còn lại ta suy ra dpcm.

II.31)Đường tròn Miquel của tứ giác toàn phầnKết quả:Cho tứ giác toàn phần Khi đó điểm và tâm của các đường tròn ngoại tiếp tam giác và cùng nằm trên 1 đường tròn - đường tròn

của tứ giác toàn phần.

145

Page 146: Tong Hop Cac Dinh Li Hinh Hoc Phang Trong Cac Ki ThiHSG

Chỉ dẫn chứng minh:Gọi và lần lượt là tâm đường tròn ngoại tiếp các tam giác và

.Gọi lần lượt là chân các đường vuông góc kẻ từ tới và Do là trung điểm của nên chúng thẳng hàng.Theo định lí đảo về đường thẳng Simson ta có cùng nằm trên 1 đường tròn.Tương tự ta suy ra dpcm.

(Thêm phần tứ giác toàn phần kèm theo, File BDF)

II.32)Hình bình hành Varignon của tứ giác .

Kết quả:Cho tứ giác ABCD có M,N,P,Q lần lượt là trung điểm của AB,BC,CD,DA.Khi đó M,N,P,Q là bốn đỉnh của một hình bình hành gọi là hình bình hành Varignon của tứ giác ABCD.

Chỉ dẫn chứng minh:

146

Page 147: Tong Hop Cac Dinh Li Hinh Hoc Phang Trong Cac Ki ThiHSG

Chứng minh kết quả này khá đơn giản,dễ thấy MN,PQ tương ứng là đường trung bình của các tam

giác ABC và ACD thế nên .Do vậy MNPQ là một hình bình hành.

I.71)Định lí Poncelet về bán kính đường tròn nội tiếp,bàng tiếp trong tam giác vuông.

Định lí:Cho tam giác có lần lượt là bán kính các đường tròn nội tiếp, bàng tiếp góc .Chứng minh rằng: tam giác ABC vuông tại A khi và chỉ khi

.

Chứng minh::Ta có:

tam giác vuông tại (dpcm)

147